Download as pdf or txt
Download as pdf or txt
You are on page 1of 702

COMPLETE

Review for
the USMLE
Step 2
Intro Gastrointestinal

In this video, we will cover:


-all GI concepts for the USMLE
Step 2!
-200 Q's on GI pathology, surgery,
pediatrics, etc.
-with awesome mnemonics!
Q1 Gastrointestinal
How is achalasia diagnosed?

A) biopsy
B) endoscopy
C) barium swallow + manometry
Q1 Gastrointestinal
How is achalasia diagnosed?

A) biopsy
B) endoscopy
C) barium swallow + manometry
Q1 Gastrointestinal
Barium Swallow Manometry
Q1 Gastrointestinal
How is achalasia diagnosed?
A) biopsy
-esophageal biopsy only for cancer, Barrett esophagus, & EoE!!
B) endoscopy
-only for excluding malignancy; achalasia shows normal mucosa
C) barium swallow + manometry
-barium swallow shows "bird's beak" sign as the esophagus
comes to a point; manometry shows failure of LES to relax
Q1 Gastrointestinal
How is achalasia diagnosed?
A) biopsy
-esophageal biopsy only for cancer, Barrett esophagus, & EoE!!
B) endoscopy
-only for excluding malignancy; achalasia shows normal mucosa
C) barium swallow + manometry
-barium swallow shows "bird's beak" sign as the esophagus
comes to a point; manometry shows failure of LES to relax
Q1 Gastrointestinal
Symptoms of Achalasia
-progressive dysphagia (both solids and liquids), chest pain, regurgitation
of undigested food, weight loss, cough
Diagnosis of Achalasia
-barium swallow, manometry, EGD to rule out obstruction/cancer
Treatment for Achalasia
-pneumatic dilation (mechanically enlarges the esophagus; very effective,
but can cause perforation)
-surgical sectioning/myotomy
-botulinum toxin injection to relax the LES; reinjection required every 3-6
months
Q1 Gastrointestinal

By the way...

Barium swallow = Upper GI series = Esophogram


Q2 Gastrointestinal
A 65-year-old woman complains of intermittent sudden
severe chest pain. Barium study is shown. What is the
diagnosis?

A) achalasia
B) esophageal spasm
Q2 Gastrointestinal
A 65-year-old woman complains of intermittent sudden
severe chest pain. Barium study is shown. What is the
diagnosis?

A) achalasia
B) esophageal spasm
Q2 Gastrointestinal
A 65-year-old woman complains of intermittent sudden
severe chest pain. Barium study is shown. What is the
diagnosis?

A) achalasia - not a "corkscrew"


B) esophageal spasm - seen in older patients
Q2 Gastrointestinal
Esophageal spasm
-motility disorder in which normal peristalsis is periodically
interrupted by high-amplitude, non-peristaltic contractions
-presents with heartburn, chest pain (but normal EKG and stress
test), dysphagia, and odynophagia
-diagnosis: barium swallow showing "corkscrew" esophagus;
manometry (most accurate) showing high amplitude contractions
-tx: CCB's, TCA's, nitrates (*careful!*); PPI's in some cases
Q2 Gastrointestinal

What autoimmune condition is


associated with esophageal
aperistalsis??
Q2 Gastrointestinal

What autoimmune condition is


associated with esophageal
aperistalsis??

SCLERODERMA!
Q2 Gastrointestinal
What autoimmune condition is
associated with esophageal
aperistalsis??

SCLERODERMA!
C.R.E.S.T. = calcinosis, Raynaud, esophageal dysmotility,
sclerodactyly, telangectasias
Q3 Gastrointestinal
A 51-year-old lady with a BMI of 41 complains of heart burn,
chest pain, and dysphagia. Endoscopy reveals a hiatal hernia.
What is the initial treatment?

A) Weight loss and PPI's


B) Surgery
Q3 Gastrointestinal
A 51-year-old lady with a BMI of 41 complains of heart burn,
chest pain, and dysphagia. Endoscopy reveals a hiatal hernia.
What is the initial treatment?

A) Weight loss and PPI's


B) Surgery
Q3 Gastrointestinal
A 51-year-old lady with a BMI of 41 complains of heart burn,
chest pain, and dysphagia. Endoscopy reveals a hiatal hernia.
What is the initial treatment?

A) Weight loss and PPI's - best initial treatment


B) Surgery - surgery usually reserved for persistence of
symptoms after medication fails; also, paraesophageal hiatal
hernias (5-10% of hiatal hernias) often require surgery to prevent
gastric volvulus
Q3 Gastrointestinal
CXR showing sliding hiatal hernia
Q4 Gastrointestinal
A 34-year-old woman recently diagnosed with AIDS comes to the
ED with pain on swallowing that has progressively gotten worse.
The patient is not on any medications and the CD4 count = 39.
What is the next step?

A) Oral nystatin
B) IV Amphotericin
C) Oral fluconazole
Q4 Gastrointestinal
A 34-year-old woman recently diagnosed with AIDS comes to the
ED with pain on swallowing that has progressively gotten worse.
The patient is not on any medications and the CD4 count = 39.
What is the next step?

A) Oral nystatin
B) IV Amphotericin
C) Oral fluconazole
Q4 Gastrointestinal
A 34-year-old woman recently diagnosed with AIDS comes to the
ED with pain on swallowing that has progressively gotten worse.
The patient is not on any medications and the CD4 count = 39.
What is the next step?

A) Oral nystatin - oral candidiasis, not esophagus


B) IV Amphotericin - 2nd line for confirmed candidiasis
C) Oral fluconazole - although CMV and herpes can cause
esophageal infection, 90% of esophageal infections in AIDS
patients are Candida; oral fluconazole is the best option
Q5 Gastrointestinal
Which medication can cause esophagitis?

A) Alendronate
B) Tetracyclines
C) NSAIDs
D) KCl
E) All of the above
Q5 Gastrointestinal
Which medication can cause esophagitis?

A) Alendronate
B) Tetracyclines
C) NSAIDs
D) KCl
E) All of the above
Q5 Gastrointestinal
Which medication can cause esophagitis?

A) Alendronate
B) Tetracyclines
C) NSAIDs
D) KCl
E) All of the above - *I KANT eat!*
KCl, alendronate, NSAIDs, Tetracyclines
Q5 Gastrointestinal
Which medication can cause esophagitis?

A) Alendronate
B) Tetracyclines
C) NSAIDs
D) KCl
E) All of the above - tx: stop the offending drug!
Q5 Gastrointestinal
Typical symptoms of
pill esophagitis include
sudden-onset
odynophagia and
retrosternal pain that
can sometimes cause
difficulty swallowing.
Q6 Gastrointestinal
A 40-year-old woman complains of a burning sensation
on her tongue along with difficulty swallowing. Labs
show microcytic anemia. Which of the following is
associated with her underlying condition?

A) obesity
B) esophageal webs
Q6 Gastrointestinal
A 40-year-old woman complains of a burning sensation
on her tongue along with difficulty swallowing. Labs
show microcytic anemia. Which of the following is
associated with her underlying condition?

A) obesity
B) esophageal webs
Q6 Gastrointestinal
Plummer-Vinson Syndrome triad

1) microcytic anemia
2) esophageal web
3) dysphagia
(iron replacement may lead to resolution of the lesion)

mnemonic: the plumber *M.A.D.E.* a boo-boo!


Micro. Anemia, Dysphagia, Eso-web
-increased risk for esophageal cancer!
Q7 Gastrointestinal
A 51-year-old man has dysphagia, halitosis, and
regurgitation of food particles. How is his underlying
condition diagnosed?

A) upper endoscopy
B) barium swallow
Q7 Gastrointestinal
A 51-year-old man has dysphagia, halitosis, and
regurgitation of food particles. How is his underlying
condition diagnosed?

A) upper endoscopy
B) barium swallow
Q7 Gastrointestinal
A 51-year-old man has dysphagia, halitosis, and
regurgitation of food particles. How is his underlying
condition diagnosed?

A) upper endoscopy - can cause perforation


B) barium swallow - used to diagnose Zenker
diverticulum, in which there is an outpocketing of the
posterior pharyngeal constrictor muscles
Q7 Gastrointestinal

Zenker
diverticulum
Q7 Gastrointestinal

Zenker
diverticulum
(don't do upper endoscopy!
may cause perforation in
patients with ZD!)
Q8 Gastrointestinal
After vomiting all day, a 43-year-old man vomits more -
but this time, bright red blood can be seen in the vomit.
The patient has no pain nor dysphagia. What is the
treatment?

A) orange juice
B) no treatment (self-resolving)
Q8 Gastrointestinal
After vomiting all day, a 43-year-old man vomits more -
but this time, bright red blood can be seen in the vomit.
The patient has no pain nor dysphagia. What is the
treatment?

A) orange juice
B) no treatment (self-resolving)
Q8 Gastrointestinal
After vomiting all day, a 43-year-old man vomits more -
but this time, bright red blood can be seen in the vomit.
The patient has no pain nor dysphagia. What is the
treatment?

A) orange juice - better to avoid acidic foods


B) no treatment (self-resolving)
Mallory-Weiss is a non-penetrating tear of the mucosa; resolves in 1-2 weeks (in case
of persistent bleeding, epinephrine or cauterization can help)
Q8 Gastrointestinal
Q9 Gastrointestinal
Esophageal cancer is accurately diagnosed with:

A) Barium study
B) EGD with biopsy
Q9 Gastrointestinal
Esophageal cancer is accurately diagnosed with:

A) Barium study
B) EGD with biopsy
Q9 Gastrointestinal
Esophageal cancer is accurately diagnosed with:

A) Barium study - best initial test; shows irregular


mass protruding into the lumen
B) EGD with biopsy - required to establish a diagnosis
(get CT scan and ultrasound for tumor staging)
Q9 Gastrointestinal
Esophageal cancer
-SCC (upper 2/3) is the most common esophageal cancer around the
world; risk factors - alcohol, tobacco
-Adenocarcinoma (lower 1/3) is the most common esophageal
cancer in America and Europe; risk factor - Barret esophagus

~Early metastasis because esophagus has no serosa;


TREAT WITH CHEMO & SURGICAL RESECTION!~
No resection = No cure
Q10 Gastrointestinal
What is depicted in this image taken during upper endoscopy?

A) GERD
B) Esophageal cancer
Q10 Gastrointestinal
What is depicted in this image taken during upper endoscopy?

A) GERD
B) Esophageal cancer
Q10 Gastrointestinal
What is depicted in this image taken during
upper endoscopy?
A) GERD - columnar metaplasia at the lower
esophagus

B) Esophageal cancer - a mass protrudes into


the lumen
Q10 Gastrointestinal
GERD
-reflux of gastric contents into the esophagus, most commonly from
transient LES relaxation (also from incompetent LES, hiatal hernia, or
gastroparesis)
-diagnosed clinically (if uncertain, most accurate test is 24-hour pH
monitoring); if refractory to empiric therapy, can get a biopsy to rule out
Barret esophagus & cancer
-Treatment: lifestyle mod. (weight loss, avoid nighttime eating, avoid
alcohol & coffee [decrease LES tone], antacids (e.g., cimetidine), H2
blockers (e.g., omeprazole), PPI's, surgery if severe
Q10 Gastrointestinal
GERD
A 48-year-old man comes to the office with several weeks of epigastric
pain radiating up his chest which is worse after lying flat for an hour. He
also has a bad taste in his mouth and a sore throat. What is the next
step?

A) Cimetidine
B) Antacid
C) Lansoprazole
D) Endoscopy
Q10 Gastrointestinal
GERD
A 48-year-old man comes to the office with several weeks of epigastric
pain radiating up his chest which is worse after lying flat for an hour. He
also has a bad taste in his mouth and a sore throat. What is the next
step?

A) Cimetidine - H2 blockers aren't as good as PPI's


B) Antacid - PPI's are first line for GERD
C) Lansoprazole - a PPI; controls symptoms of GERD
D) Endoscopy - worthless for GERD; unnecessary anyway
Q11 Gastrointestinal
What are causes of acute gastritis?

A) NSAIDs
B) Alcohol
C) H pylori
D) Toxic ingestion
E) All of the above
Q11 Gastrointestinal
What are causes of acute gastritis?

A) NSAIDs
B) Alcohol
C) H pylori
D) Toxic ingestion
E) All of the above
Q11 Gastrointestinal
Acute Gastritis
-symptoms include epigastric pain, nausea, vomiting,
hematemesis, GI bleed; may be asymptomatic
-upper endoscopy required for diagnosis!!
-Tx: stop offending agent; antacids, H2 blockers,
sucralfate may help - not so helpful though; triple therapy
for H pylori (amoxacillin [or metronidazole if allergy],
clarithromycin, omeprazole) [triple *ACO*]
Q12 Gastrointestinal
Chronic gastritis is most commonly caused by:

A) Autoantibodies to parietal cells


B) H pylori & NSAIDS
Q12 Gastrointestinal
Chronic gastritis is most commonly caused by:

A) Autoantibodies to parietal cells


B) H pylori & NSAIDS
Q12 Gastrointestinal
Chronic gastritis is most commonly caused by:

A) Autoantibodies to parietal cells - 10% of the


time; a.k.a. type A; occurs in the fundus
B) H pylori & NSAIDS - 90% of the time; a.k.a.,
type B; occurs in the antrum
Q12 Gastrointestinal
Chronic gastritis is most commonly caused by:

A) Autoantibodies to parietal cells - 10% of the


time; a.k.a. type A; occurs in the fundus
B) H pylori & NSAIDS - 90% of the time; a.k.a.,
type B; occurs in the antrum *"Ant Pylori"*
Q13 Gastrointestinal
What is the most accurate test for H pylori:

A) Biopsy
B) Serology
C) Breath test
D) Stool antigen
Q13 Gastrointestinal
What is the most accurate test for H pylori:

A) Biopsy
B) Serology
C) Breath test
D) Stool antigen
Q13 Gastrointestinal
What is the most accurate test for H pylori:

A) Biopsy - requires endoscopy


B) Serology - not specific (maybe old infection)
C) Breath test - positive only in active infection
D) Stool antigen - requires stool sample
(but both breath test and stool antigen are very sensitive and
specific)
Q14 Gastrointestinal
An enlarged left supraclavicular lymph node is
associated with:

A) Gastric cancer
B) Testicular cancer
Q14 Gastrointestinal
An enlarged left supraclavicular lymph node is
associated with:

A) Gastric cancer
B) Testicular cancer
Q14 Gastrointestinal
An enlarged left supraclavicular lymph node is
associated with:

A) Gastric cancer - Virchow node


(also, Sister Mary Joseph node is a palpable lymph node
near the umbilicus; also ass. w/ gastric cancer)
B) Testicular cancer
Q14 Gastrointestinal

Virchow node

Sister Mary Joseph node


Q14 Gastrointestinal
Gastric cancer
-malignant tumor with a poor prognosis
-usually discovered late because early disease is
asymptomatic; may be loss of appetite and
indigestion; GI bleed later on
-Seen in Korea and Japan (high salt/nitrites)
-Diagnosis: upper endoscopy with biopsy
Q14 Gastrointestinal
Gastric cancer
-if metastases to the ovary it is called...
Q14 Gastrointestinal
Gastric cancer
-if metastazes to the ovary it is called...
A KRUKENBERG TUMOR!
Q14 Gastrointestinal
Gastric cancer
-if metastazes to the ovary it is called...
A KRUKENBERG TUMOR!

CRO
OK!
Q15 Gastrointestinal
After a meal, pain caused by gastric ulcer is:

A) Greater
B) Decreased
Q15 Gastrointestinal
After a meal, pain caused by gastric ulcer is:

A) Greater
B) Decreased
Q15 Gastrointestinal
After a meal, pain caused by gastric ulcer is:

A) Greater - Gastric gets GREATER


B) Decreased - Duodenal gets DECREASED
Q15 Gastrointestinal
After a meal, pain caused by gastric ulcer is:

A) Greater - Gastric gets GREATER


B) Decreased - Duodenal gets DECREASED

but this fact is not specific enough to make a


diagnosis of GU vs. DU!
Q15 Gastrointestinal
Peptic (Gastric & Duodenal) Ulcer
-results from damage to gastric/duodenal mucosa
-caused by impaired mucosal defense or increase in acidic gastric
contents
-risk factors: similar to gastritis (H pylori, NSAIDs, alcohol, tobacco)
-presents with epigastric pain, nausea, hematemesis, melena
-most accurate test: upper endoscopy with biopsy
-tx: antacids, PPI's, or H2 blockers; triple therapy for H pylori;
discontinue offending agents; surgery if severe or perforation
Q15 Gastrointestinal
What do you do if NSAIDs are causing ulcers
but the person needs NSAIDS (e.g., for
arthritis)???
Q15 Gastrointestinal
What do you do if NSAIDs are causing ulcers
but the person needs NSAIDS (e.g., for
arthritis)???

Misoprostol!
"My poor ulcer - so long!"
Q16 Gastrointestinal
A man comes in with recurring gnawing, burning,
abdominal pain; he has diarrhea, nausea, vomiting,
weight loss, and HYPERCALCEMIA. What is the
diagnosis?

A) ZES
B) Not sure
Q16 Gastrointestinal
A man comes in with recurring gnawing, burning,
abdominal pain; he has diarrhea, nausea, vomiting,
weight loss, and HYPERCALCEMIA. What is the
diagnosis?

A) ZES
B) Not sure
Q16 Gastrointestinal
A man comes in with recurring gnawing, burning,
abdominal pain; he has diarrhea, nausea, vomiting,
weight loss, and HYPERCALCEMIA. What is the
diagnosis?

A) ZES - a rare condition; gastrin producing tumor in


the duodenum and/or pancreas; excess gastrin
secretion leads to recurrent ulcers; hypercalcemia from
hyperparathyroidism (e.g., MEN type 1)
Q16 Gastrointestinal
Diagnosis of ZES
-imaging shows GI ulcers
-get serum gastrin levels (off PI therapy for a week)
-if gastrin levels high (>1000) or nondiagnostic (110-
1000), get secretin stimulation test
-secretin inhibits gastrin release by normal gastric G
cells, but stimulates gastrin release by gastrinoma cells -
so a rise in gastrin would indicate gastrinoma

Tx: high dose PPI, surgical resection if no metastasis


Q17 Gastrointestinal
A man who owns dog has the following image of
his liver. What is the diagnosis?
A) Campylobacter
B) C. difficile
C) Echinococcus
D) Entamoeba
E) Taenia solium
Q17 Gastrointestinal
A man who owns dog has the following image of
his liver. What is the diagnosis?
A) Campylobacter
B) C. difficile
C) Echinococcus
D) Entamoeba
E) Taenia solium
Q17 Gastrointestinal
Echinococcus - contracted from close
contacts to dogs, which is the host for the
tapeworm; causes liver cysts, often found
incidentally; may cause RUQ pain; "eggshell"
calcification on CT scan; rupture may cause
anaphylactic shock; surgical resection often
required; prolonged albendazole may work
Q17 Gastrointestinal
Infectious Diarrhea: Causes & Treatments
-Campylobacter - supportive tx, then flouroquinolone or azith.
-C difficile - treat with metronidazole or vanco. (severe)
-E granulosus - surgical resection; albendazole
-E histolytica - metronidazole
-E coli 1057:H7 - supportive care; avoid drugs which may cause HUS
-Salmonella - fluids; tx bacteremia or SS patients with floro or TMP
-Shigella - TMP-SMX to reduce person to person spread
-Taenia solium - albendazole and symptomatic tx of CNS symptoms
-Trichinella - albendazole; no tx: once penetration to muscles
Q18 Gastrointestinal
What is the initial test for a patient with suspected
celiac disease?

A) tTG-IgA Test
B) Biopsy
Q18 Gastrointestinal
What is the initial test for a patient with suspected
celiac disease?

A) tTG-IgA Test
B) Biopsy
Q18 Gastrointestinal
What is the initial test for a patient with suspected
celiac disease?

A) tTG-IgA Test - very sensitive (alternative is the


anti-endomysial antibody test)
B) Biopsy - gold standard
Q18 Gastrointestinal
Celiac disease
-pale, foul smelling bulky stools, abdominal pain,
flatus, bloating, weight loss, fatigue
-10% of patients get Dermatitis herpetiformis
(grouped papulovesicular lesions on elbows, knees,
butt, or posterior scalp)
-tx: gluten free diet
Q19 Gastrointestinal
Malabsorption + arthritis + lymphadenopathy +
PAS positive granules in lamina propria on biopsy
=

???
Q19 Gastrointestinal
Malabsorption + arthritis + lymphadenopathy +
PAS positive granules in lamina propria on biopsy
=

Whipple disease
Q19 Gastrointestinal
Malabsorption + arthritis + lymphadenopathy +
PAS positive granules in lamina propria on biopsy
=

Whipple disease
(treat with long term antibiotics)
Q20 Gastrointestinal
How is lactose intolerance diagnosed?

A) Breath test
B) No test
Q20 Gastrointestinal
How is lactose intolerance diagnosed?

A) Breath test
B) No test
Q20 Gastrointestinal
How is lactose intolerance diagnosed?

A) Breath test - hydrogen breath test shows


increased hydrogen after ingestion of lactose
B) No test
Q20 Gastrointestinal
Lactose intolerance
-presents with abdominal bloating,
flatulence, cramping, and watery diarrhea
after ingesting dairy products
-Tx: avoidance of dairy products; oral lactase
enzyme replacement
Q21 Gastrointestinal
What is the best initial test for Carcinoid
syndrome?

A) CT scan and In-111 octreotide scan


B) Urine levels of 5-HIAA
Q21 Gastrointestinal
What is the best initial test for Carcinoid
syndrome?

A) CT scan and In-111 octreotide scan


B) Urine levels of 5-HIAA
Q21 Gastrointestinal
What is the best initial test for Carcinoid
syndrome?

A) CT scan and In-111 octreotide scan -


used to localize the tumor
B) Urine levels of 5-HIAA- since the
metastasized tumor releases serotonin
Q21 Gastrointestinal
Carcinoid syndrome
-caused by metatasis of carcinoid tumors (commonly
arise from the ileum and appendix); produces serotonin;
w/o metastasis, hormones are metabolized in the liver;
no systemic symptoms
-symptoms: flushing, diarrhea, abdominal cramps, right-
sided cardiac valvular lesions
-tx: octreotide and surgical resection
Q22 Gastrointestinal
What vitamin deficiency do patients with
carcinoid syndrome develop

A) Vitamin A deficiency
B) Vitamin B3 deficiency
C) Vitamin D deficiency
Q22 Gastrointestinal
What vitamin deficiency do patients with
carcinoid syndrome develop

A) Vitamin A deficiency
B) Vitamin B3 deficiency
C) Vitamin D deficiency
Q22 Gastrointestinal
What vitamin deficiency do patients with carcinoid
syndrome develop

Vitamin B3 deficiency
-patients get vitamin b3 (i.e., niacin) deficiency (pellagra-
Diarrhea, Dermititis, Dementia) because the tumor uses
tryptophan to produce serotonin
Q23 Gastrointestinal
The most common cause of SBO in adults
is:

A) hernias
B) adhesions (e.g., previous surgery)
Q23 Gastrointestinal
The most common cause of SBO in adults
is:

A) hernias
B) adhesions (e.g., previous surgery)
Q23 Gastrointestinal
The most common cause of SBO in adults
is:

A) hernias
B) adhesions (e.g., previous surgery)
-adhesions in adults; hernias in children
Q23 Gastrointestinal
The most common cause of SBO in adults
is:

A) hernias
B) adhesions (e.g., previous surgery)
-adhesions in adults; hernias in children
~what's gallstone ileus?~
Q23 Gastrointestinal

The most common cause of SBO in adults


is:
A) hernias
B) adhesions (e.g., previous surgery)
-adhesions in adults; hernias in children
~gallstone ileus - SBO caused by a gallstone that lodges
at the ileocecal valve
Q23 Gastrointestinal
Small Bowel Obstruction
-causes: adhesions (60%), hernias (20%)
-in partial SBO, the patient passes gas, but no stool; in
complete SBO, there is no gas or stool
-crampy abdominal pain at 5 minute intervals; vomiting often
follows the pain; abdominal distention
-complications: ischemic necrosis (lactic acidosis), peritonitis,
rupture
-diagnosis: first x-ray; CT more accurate
Q23 Gastrointestinal
Small Bowel Obstruction - TREATMENT
-best initial: fluid resuscitation
-partial: supportive care (NPO, NG suction, IV hydration, pain
management, etc.) may be sufficient
-complete obstruction - exploratory laparotomy
Q24 Gastrointestinal

Which condition is associated with the


development of ileus?

A) Diabetes mellitus
B) Hyperkalemia
Q24 Gastrointestinal

Which condition is associated with the


development of ileus?

A) Diabetes mellitus
B) Hyperkalemia - HYPOkalmia
Q24 Gastrointestinal
Ileus
-loss of peristalsis without structural obstruction
-causes: recent surgery, medical illness, hypokalemia, other electrolyte
abnormality, hypothyroidism, DM
-presentation: diffuse constant abdominal pain, nausea and vomiting,
and an absence of flatus or bowel movements
-dx: best initial test - xray (film shows distended loops with air seen
throughout the colon and rectum; vs. SBO - no air distal to the
obstruction)
-tx: bowel rest, discontinue narcotics or other drugs that reduce bowel
motility; NG suction/parental feeds as needed
Q25 Gastrointestinal

What is the most accurate test for mesenteric


ischemia?

A) CT scan
B) CT angiography
Q25 Gastrointestinal

What is the most accurate test for mesenteric


ischemia?

A) CT scan- best initial test (air in bowel)


B) CT angiography- gold standard for
diagnosis of arterial occlusive disease
Q25 Gastrointestinal
Mesenteric ischemia
-insufficient blood supply to the small intestine, resulting in ischemia
and potentially necrosis
-caused by emboli (most commonly from the heart; e.g., afib), arterial
occlusion (risk factor: atherosclerosis), venous thrombosis
-presents with severe abdominal pain out of proportion to the exam,
nausea, vomiting, diarrhea, bloody stools
-tx: volume resuscitation, broad spectrum antibiotics, anticoagulation
(resection of infarcted bowel)
Q26 Gastrointestinal

What is the most common cause of lower GI


bleed in patients >40?

A) Hemorrhoids
B) Diverticulosis
Q26 Gastrointestinal

What is the most common cause of lower GI


bleed in patients >40?

A) Hemorrhoids
B) Diverticulosis
Q26 Gastrointestinal

What is the most common cause of lower GI


bleed in patients >40?

A) Hemorrhoids - 2nd to most common


B) Diverticulosis - most common
Q26 Gastrointestinal
Diverticulosis
-outpouching of the mucosa and submucosa that herniate though
the colonic muscle layers; most common in sigmoid colon
-risk factors: low-fiber and high fat diet; older age, connective
tissue disorders
-often asymptomatic until patients present with sudden
intermittent painless bleeding; can lead to diverticulitis
(inflammation of a diverticulum 2nd to fecalith impaction)
-dx: colonoscopy (diverticulosis); CT scan (diverticulitis)
Q26 Gastrointestinal
Q27 Gastrointestinal

What is the most common outcome of LBO?

A) Self-resolution
B) Surgery
Q27 Gastrointestinal

What is the most common outcome of LBO?

A) Self-resolution
B) Surgery
Q27 Gastrointestinal
Large Bowel Obstruction
presentation: significant distention, tympany and tenderness;
fever and signs of shock suggest perforation/peritonitis or
ischemia/necrosis
-causes: colon cancer (most common cause), diverticulitis,
volvulus, fecal impaction, benign tumors
-RULE: ASSUME COLON CANCER UNTIL PROVEN
OTHERWISE!
Q28 Gastrointestinal

A patient with a diagnosis of aortic valve


endocarditis is given IV antibiotics; cultures are
positive for S. bovis. With is the next step?
Q28 Gastrointestinal

A patient with a diagnosis of aortic valve


endocarditis is given IV antibiotics; cultures are
positive for S. bovis. With is the next step?
Colonscopy!
Q28 Gastrointestinal

A patient with a diagnosis of aortic valve


endocarditis is given IV antibiotics; cultures are
positive for S. bovis. With is the next step?
Colonscopy!
-although the mechanism is not well understood,
there is significant association between S. bovis
and colon cancer
Q29 Gastrointestinal

Most patients with colorectal cancer are


asymptomatic. T/F

A) True
B) False
Q29 Gastrointestinal

Most patients with colorectal cancer are


asymptomatic. T/F

A) True
B) False
Q29 Gastrointestinal

Most patients with colorectal cancer are


asymptomatic. T/F

A) True - symptoms include weight loss, anorexia,


blood in stool; obstruction in left sided lesions; bright
red blood in rectal lesions
B) False
Q30 Gastrointestinal

Ischemic colitis most commonly affects:

A) the right colon


B) the left colon
Q30 Gastrointestinal

Ischemic colitis most commonly affects:

A) the right colon


B) the left colon
Q30 Gastrointestinal
Ischemic colitis most commonly affects:

A) the right colon


B) the left colon
most commonly at the splenic flexure by the "watershed area"; usually occurs in
the setting of atherosclerosis.
-presents with lower abdominal pain followed by bloody diarrhea after meals,
exertion, or in the heat; fever may suggest bowel necrosis
-tx: bowel rest, fluids, antibiotics; surgical resection in infarction, obstruction, or
fulminant colitis
Q30 Gastrointestinal
Q31 Gastrointestinal

An upper GI bleed in classified as one above:

A) the colon
B) the jejunum
Q31 Gastrointestinal

An upper GI bleed in classified as one above:

A) the colon
B) the jejunum
Q31 Gastrointestinal

An upper GI bleed in
classified as one above:

A) the colon
B) the jejunum - upper GI bleed is
classified as a bleed proximal to the
ligament of Treitz, which separates the
duodenum & jejunum
Q31 Gastrointestinal

Upper GI Bleeds: more often melena

Lower GI bleeds: more often hematochezia


Q31 Gastrointestinal

Upper GI Bleeds: more often melena


dx: NG tube and lavage; endoscopy

Lower GI bleeds: more often hematochezia


dx: anoscopy/sigmoidoscopy
Q31 Gastrointestinal

But remember! Give fluids comes BEFORE trying to


figure out the etiology!!
Q32 Gastrointestinal

Transmural inflammation is seen in:

A) Ulcerative colitis
B) Crohn disease
Q32 Gastrointestinal

Transmural inflammation is seen in:

A) Ulcerative colitis
B) Crohn disease
Q32 Gastrointestinal
Transmural inflammation is seen in:

A) Ulcerative colitis - inflammation and ulceration are limited


to the mucosa and submucosa; rectum always involved;
continuous lesions
B) Crohn disease - transmural inflammation; sometimes
leading to fistulas to other organs; "skip lesions"
Q32 Gastrointestinal
Treatment for Ulcerative Colitis & Crohn
-5-ASA agents (e.g., sulfasalazine, mesalamine)
-steroids for flare ups
-immunomodulators (e.g., azathioprine)
-biologics (e.g., infliximab) for refractory or mild/severe
disease
-surgery may be requried: total proctocolectomy (UC) or
select surgical resection (Crohn's disease)
Q33 Gastrointestinal

Indirect hernias involve herniation of abdominal


contents (medial/lateral) to the inferior epigastric
vessels.

A) medial
B) lateral
Q33 Gastrointestinal

Indirect hernias involve herniation of abdominal


contents lateral to the inferior epigastric vessels.

A) medial
B) lateral
Q33 Gastrointestinal

Youtube:
@AJmonics
Q34 Gastrointestinal

What condition is this triad associated with?

A) Cholelithiasis
B) Cholecystitis
C) Choledocholithiasis
D) Cholangitis
Q34 Gastrointestinal

What condition is this triad associated with?

A) Cholelithiasis
B) Cholecystitis
C) Choledocholithiasis
D) Cholangitis
Q34 Gastrointestinal

What condition is this triad associated with?

A) Cholelithiasis - gallstones
B) Cholecystitis - gallbladder inflammation
C) Choledocholithiasis - stone in the CBD
D) Cholangitis - infection of the CBD (e.g., stone)
Q34 Gastrointestinal

What condition is this triad associated with?

A) Cholelithiasis - gallstones
B) Cholecystitis - gallbladder inflammation
C) Choledocholithiasis - stone in the CBD
D) Cholangitis - infection of the CBD (e.g., stone)
Reynolds pentad= + septis shock + AMS
Q34 Gastrointestinal
A) Cholelithiasis - gallstones
-dx: ultrasound

B) Cholecystitis - gallbladder inflammation


dx: ultrasound, HIDA scan

C) Choledocholithiasis - stone in the CBD


-dx: MRCP and ERCP (ultrasound doesn't show stone, but dilated CBD)

D) Cholangitis - infection of the CBD (e.g., stone)


-dx: clinical, confirmed by biliary dilation on imaging; or ERCP
Q34 Gastrointestinal
A) Cholelithiasis - gallstones
-dx: ultrasound
-tx: if symptomatic, laparoscopic cholecystectomy
B) Cholecystitis - gallbladder inflammation
dx: ultrasound, HIDA scan
-tx: laparoscopic cholecystectomy (if too ill, cutaneous drainage)
C) Choledocholithiasis - stone in the CBD
-dx: MRCP and ERCP (ultrasound doesn't show stone, but dilated CBD)
-tx: ERCP to remove stone, followed by cholecystectomy
D) Cholangitis - infection of the CBD (e.g., stone)
-dx: clinical, confirmed by biliary dilation on imaging; or ERCP
-tx: ERCP; surgery if patient is toxic
Q35 Gastrointestinal

What is the treatment for gallstone illeus?

A) reassurance
B) laparotomy with stone extraction
Q35 Gastrointestinal

What is the treatment for gallstone illeus?

A) reassurance
B) laparotomy with stone extraction
Q36 Gastrointestinal

Elevated ALP and bilirubin is a sign of:

A) Hepatocellular injury
B) Cholestasis
Q36 Gastrointestinal

Elevated ALP and bilirubin is a sign of:

A) Hepatocellular injury
B) Cholestasis
Q36 Gastrointestinal

Elevated ALP and bilirubin is a sign of:

A) Hepatocellular injury - ↑AST & ↑ALT


B) Cholestasis - ↑ALP & ↑bilirubin
Q36 Gastrointestinal

Elevated ALP and bilirubin is a sign of:

A) Hepatocellular injury - ↑AST & ↑ALT


B) Cholestasis - ↑ALP & ↑bilirubin

(Mixed: a combination of both)


Q37 Gastrointestinal

The most common causes of chronic viral


hepatitis are:

A) HAV, HDV
B) HBV, HCV
Q37 Gastrointestinal

The most common causes of chronic viral


hepatitis are:

A) HAV, HDV
B) HBV, HCV
Q37 Gastrointestinal

The most common causes of chronic viral


hepatitis are:

A) HAV, HDV
B) HBV, HCV - **"b/c" it's chronic!**
Q37 Gastrointestinal
Acute Hepatitis
-Causes: viruses (HAV, HBV, HCV, HDV, HEV), alcohol, and drugs (e.g.,
Tylenol, methyldopa)
-fulminant (aka acute liver failure); severe liver injury with INR > 1.5 and
hepatic encephalopathy in a patient without underlying chronic liver
disease
-history: nonspecific viral prodrome (e.g., fever, nausea), followed by
jaundice and RUQ pain
-diagnosis: ↑ALT, ↑AST, ↑ALP,↑GGT, ↑bilirubin
-treatment: supportive care; acute HCV is only one that's treated (e.g.,
sofosbuvir + ledipasvir or sofosbuvir + velpatasvir)
Q37 Gastrointestinal
Chronic Hepatitis
-Causes: most commonly viral infection (HCV in the US; HBV
worldwide), alcohol, autoimmune hepatitis, & metabolic syndromes
(Wilson disease, hemochromatosis, alpha-1 ATD)
-symptoms: may be asymptomatic; fatigue, muscle pains; portal
hypertension and jaundice usually only when disease has
progressed to cirrhosis
-diagnosis: ALT and AST even be normal/low for months ***
-tx: tenofovir and entecavir (HBV); 2 DAAS or 1 DAA + ribavirin
(HCV); liver transplantation in end-stage renal disease
Q38 Gastrointestinal

Which HBV serologic marker is an important


indicator of transmissibility?

A) HBs
B) HBc
C) HBe
Q38 Gastrointestinal

Which HBV serologic marker is an important


indicator of transmissibility?

A) HBs
B) HBc
C) HBe
Q38 Gastrointestinal

Which HBV serologic marker is an important


indicator of transmissibility?

A) HBs - surface antigen


B) HBc - core antigen
C) HBe - "escapes" during replication; HbeAb
indicates low transmissibility
Q39 Gastrointestinal

Which value is decreased in cirrhosis?

A) PR/INR
B) bilirubin
C) albumin
Q39 Gastrointestinal

Which value is decreased in cirrhosis?

A) PR/INR
B) bilirubin
C) albumin
Q39 Gastrointestinal

Which value is decreased in cirrhosis?

A) PR/INR
B) bilirubin - liver dysfunction → ↑bilirubin
C) albumin - albumin decreased due to
decreased synthesis by hepatocytes; if albumin
below 3.5g/dL, poor prognosis
Q39 Gastrointestinal

Which value is decreased in cirrhosis?


(NOTE: these are the values used in the initial diagnostic assessment for
cirrhosis; but liver biopsy is most accurate)
A) PR/INR
B) bilirubin
C) albumin - albumin decreased due to
decreased synthesis by hepatocytes; if albumin
below 3.5g/dL, poor prognosis
Q39 Gastrointestinal

Cirrhosis
-fibrosis and nodular regeneration resulting from
chronic hepatic injury
-symptoms: asymptomatic -> symptoms (see
pic.)
-tx: slow progression; all cirrhosis patients must
get HAV, HBV, & PPSV-23 vaccines!!**
Q39 Gastrointestinal
Q39 Gastrointestinal
Cirrhosis
How is cirrhotic ascites treated? _

How is cirrhotic SBP treated? _

How is hepatic encephalopathy treated? _

How is hepatic coagulopathy treated? _


Q39 Gastrointestinal
Cirrhosis
How is cirrhotic ascites treated?
-sodium restriction, diuretics, paracentesis, shunt
How is cirrhotic SBP treated?
-IV antibiotics acutely (e.g., ceftriaxone), IV albumin
How is hepatic encephalopathy treated?
-lactulose and/or rifaximin
How is hepatic coagulopathy treated?
-FFP for acute bleeding; (vitamin K won't help)
Q40 Gastrointestinal

Which value indicates that ascites is related to


portal hypertension?

A) SAAG > 1.1


B) SAAG < 1.1
Q40 Gastrointestinal

Which value indicates that ascites is related to


portal hypertension?

A) SAAG > 1.1


B) SAAG < 1.1
Q40 Gastrointestinal

Which value indicates that ascites is related to


portal hypertension?

A) SAAG > 1.1


B) SAAG < 1.1

-high specificity & sensitivity!


Q40 Gastrointestinal

Which value indicates that ascites is related to


portal hypertension?

A) SAAG > 1.1


[examples: splenic or portal vein thrombosis,
schistosomiasis, cirrhosis, RHF, constrictive
pericarditis, BCS]
Q41 Gastrointestinal

Primary sclerosing cholangitis is associated with:

A) P-anca and "onion skinning" sclerosis


B) autoimmunity
Q41 Gastrointestinal

Primary sclerosing cholangitis is associated with:

A) P-anca and "onion skinning" sclerosis


B) autoimmunity
Q41 Gastrointestinal

Primary sclerosing cholangitis is associated with:

A) P-anca and "onion skinning" sclerosis


B) autoimmunity - PCS is idiopathic
Q41 Gastrointestinal
Primary sclerosing cholangitis
-idiopathic disorder characterized by progressive inflammation and
fibrosis, along with strictures of extrahepatic and intrahepatic bile ducts;
usually in young men with UC; increases risk for cholangiocarcinoma
-symptoms: jaundice, pruritis, fatigue
-most accurate test: MRCP/ERCP showing bile
duct strictures and dilation ("beading");
(liver biopsy shows periductal sclerosis - "onion skinning,
**NOT the most accurate test**)
-tx: ERCP with dilation; liver transplant; UDCA can help
Q42 Gastrointestinal

Which medication slows the disease progression


of Primary biliary cholangitis?
Q42 Gastrointestinal

Which medication slows the disease progression


of Primary biliary cholangitis?

Ursodeoxycholic acid
Q42 Gastrointestinal

Which medication slows the disease progression


of Primary biliary cholangitis?

Ursodeoxycholic acid
(Cholestyramine for pruritis)
Q42 Gastrointestinal
Primary Biliary Cholangitis
-autoimmune disorder characterized by destruction of
intrahepatic bile ducts
-most common in middle aged women with other autoimmune
disorders
-presentation: jaundice, pruritis, fat soluble vitamin deficiency
(ADEK)
-labs: ↑ALP, + mitochondrial antibody
-Tx: ursodeoxycholic acid, cholestyramine, liver transplant
Q42 Gastrointestinal

What is the treatment for Non-alcoholic Fatty


Liver Disease?

A) weekly phlebotomy
B) weight loss, diet, exercise
Q42 Gastrointestinal

What is the treatment for Non-alcoholic Fatty


Liver Disease?

A) weekly phlebotomy
B) weight loss, diet, exercise
Q42 Gastrointestinal

If they present you with an obese diabetic patient


who develops hepatomegaly with increased
LFT's, and hepatitis/alcoholism is excluded, the
answer is:

INSULIN RESISTANCE!!
(w/o insulin, ↑peripheral lipolysis → free fatty acids abound!)
Q43 Gastrointestinal

What is the most common type of primary liver


cancer?

A) Hepatocellular carcinoma
B) Hepatoblastoma
Q43 Gastrointestinal

What is the most common type of primary liver


cancer?

A) Hepatocellular carcinoma
B) Hepatoblastoma
Q43 Gastrointestinal

What is the most common type of primary liver


cancer?

A) Hepatocellular carcinoma - HCC is one of the


most common cancers worldwide, but not
common in the US
Q43 Gastrointestinal
Hepatocellular Carcinoma
-in the US, risk factors are cirrhosis (from alcohol, HCV,
and steatohepatitis), chronic HBV (even w/o cirrhosis); in
developing countries, HBV, and aflatoxins
-presentation: RUQ pain, abdominal distention, liver
failure (easy bruisability, jaundice, coagulopathy)
-advanced disease can involve metastasis to the lungs,
peritoneum, or bone (e.g., the spine)
-diagnosis: mass on ultrasound/CT, abnormal LFT's,
↑↑AFP (50%); get biopsy if uncertain
Q44 Gastrointestinal

How is primary hemochromatosis inherited?

A) autosomal recessive
B) autosomal dominant
Q44 Gastrointestinal

How is primary hemochromatosis inherited?

A) autosomal recessive
B) autosomal dominant
Q44 Gastrointestinal

How is primary hemochromatosis inherited?

A) autosomal recessive - characterized by


mutations in the HFE gene that results in
excessive absorption of dietary iron (2° occurs in
chronic transfusion therapy; e.g., SCD)
B) autosomal dominant
Q44 Gastrointestinal
Hemochromatosis
-a state of iron overload
-hemosiderin accumulates in the liver, pancreas, heart, adrenal
glands, testes, and pituitary gland
-bronze skin, abdominal pain (hepatomegaly), hypogonadism,
anthropathy of finger joints, HF, impotence
-tx: weekly phlebotomy to normalize serum iron levels; and then
every 2-4 months for maintenance (also iron chelating agents
such as deferoxamine for maintenace)
Q45 Gastrointestinal
What is the gold standard for a patient with the
following eye finding?

A) ↓ceruloplasmin
B) 24-urinary copper excretion after giving
penicillamine
Q45 Gastrointestinal
What is the gold standard for a patient with the
following eye finding?

A) ↓ceruloplasmin
B) 24-urinary copper excretion after giving
penicillamine
Q45 Gastrointestinal
What is the gold standard for a patient with the
following eye finding?

A) ↓ceruloplasmin
B) 24-urinary copper excretion after giving
penicillamine - Wilson disease (KF rings)
Q45 Gastrointestinal
Wilson Disease
-an autosomal recessive disorder that results in
defective copper transport and subsequent
accumulation and deposition of copper in the
liver and brain (psychosis, delusions)
-tx: penicillamine [1st] or trientine [2nd] (copper chelators that
increase urinary copper); avoid dietary copper (e.g., shellfish,
legumes); zinc (increases fecal excretion)
Q45 Gastrointestinal
Which condition is highly malignant?

A) Insulinoma
B) VIPoma
Q45 Gastrointestinal
Which condition is highly malignant?

A) Insulinoma
B) VIPoma
Q45 Gastrointestinal
A) Insulinoma
hypoglycemia (with sweating, anxiety, headaches) that resolves with
correction of the hypoglycemia; get fasting lab results (elevated serum
insulin, elevated C-peptide); tx: surgical resection of the tumor
B) VIPoma
results from VIP-producing tumor; highly malignant; presents with watery
diarrhea, dehydration, muscle weakness, and flushing; tx: replace fluids
and electrolyte losses; surgery to resect tumor; consider octreotide
Q45 Gastrointestinal
The "chain of lakes" sign helps in the diagnosis
of:

A) acute pancreatitis
B) chronic pancreatitis
Q45 Gastrointestinal
The "chain of lakes" sign helps in the diagnosis
of:

A) acute pancreatitis
B) chronic pancreatitis -
represents alternating stenosis and dilation of the
main pancreatic duct on CT/ultrasound
Q45 Gastrointestinal
ACUTE PANCREATITIS
-leakage of activated pancreatic enzymes into
pancreatic and peripancreatic tissue
-85%-90% self limited
CHRONIC PANCREATITIS
-irreversible parenchymal destruction, leading to
pancreatic dysfunction and insufficiency
-may get chronic pain & pancreatic dysfunction
Q45 Gastrointestinal
Courvoisier sign is associated with which type of
cancer?
Q45 Gastrointestinal
Courvoisier sign is associated with which type of
cancer?

Pancreatic cancer! Refers to a palpable,


nontender gallbladder.
Q45 Gastrointestinal
Pancreatic Cancer
-75% of adenocarcinomas in the head of the pancreas; risk factors: smoking,
chronic pancreatitis, first degree relative with pancreatic cancer; more common
in men
-presentation: abdominal pain radiating to the back; obstructive jaundice (if at
head of the pancreas, due to bile duct obstruction), loss of appetite, nausea,
vomiting, weight loss, weakness, fatigue, and indigestion
-diagnosis: best initial test - CT scan with contrast; if jaundice, ultrasound; CA-
19-9 elevated but is neither sensitive nor specific
-treatment: palliative chemotherapy if advanced, whipple procedure if small in
head, pancreatomy/splenectomy if small or in body or tail
Q46 Gastrointestinal
A man with C diff is treated with vancomycin. He
develops C diff diarrhea again 2 months later. What is
the best next step?

A) Retreat with oral vancomycin


B) Oral metronidazole
C) IV metronidazole
D) IV vancomycin
Q46 Gastrointestinal
A man with C diff is treated with vancomycin. He
develops C diff diarrhea again 2 months later. What is
the best next step?

A) Retreat with oral vancomycin


B) Oral metronidazole
C) IV metronidazole
D) IV vancomycin
Q46 Gastrointestinal
A man with C diff is treated with vancomycin. He
develops C diff diarrhea again 2 months later. What is
the best next step?

A) Retreat with oral vancomycin


B) Oral metronidazole - 1st line in children
C) IV metronidazole - IV only if oral doesn't work
D) IV vancomycin - no! won't pass the bowel wall!
Q46 Gastrointestinal
A man with C diff is treated with vancomycin. He
develops C diff diarrhea again 2 months later. What is
the best next step?

A) Retreat with oral vancomycin


-and if THAT doesn't work, then switch to FIDAXOMICIN
Q47 Gastrointestinal
Which of the following is associated with the worst
prognosis in pancreatitis?

A) Elevated amylase
B) Elevated lipase
C) Pain intensity
D) Hypocalcemia
Q47 Gastrointestinal
Which of the following is associated with the worst
prognosis in pancreatitis?

A) Elevated amylase
B) Elevated lipase
C) Pain intensity
D) Hypocalcemia
Q47 Gastrointestinal
Which of the following is associated with the worst
prognosis in pancreatitis?

A) Elevated amylase
B) Elevated lipase
C) Pain intensity
D) Hypocalcemia (severe pancreatitis -> fat
malabsorption -> calcium binds fat in the bowel)
Q48 Gastrointestinal
A 27-year-old female complains of chronic diarrhea and
steatorrhea. Serum electrolytes are normal, and no microbes are
found in the stool. The D-xylose test is performed, and the
patient's urine is found to have lower than normal levels of D-
xylose. What is the diagnosis?

A) Pancreatic insufficiency
B) Celiac disease
Q48 Gastrointestinal
A 27-year-old female complains of chronic diarrhea and
steatorrhea. Serum electrolytes are normal, and no microbes are
found in the stool. The D-xylose test is performed, and the
patient's urine is found to have lower than normal levels of D-
xylose. What is the diagnosis?

A) Pancreatic insufficiency
B) Celiac disease
Q48 Gastrointestinal
A 27-year-old female complains of chronic diarrhea and
steatorrhea. Serum electrolytes are normal, and no microbes are
found in the stool. The D-xylose test is performed, and the
patient's urine is found to have lower than normal levels of D-
xylose. What is the diagnosis?

A) Pancreatic insufficiency
B) Celiac disease
Q48 Gastrointestinal
A 27-year-old female complains of chronic diarrhea and
steatorrhea. Serum electrolytes are normal, and no microbes are
found in the stool. The D-xylose test is performed, and the
patient's urine is found to have lower than normal levels of D-
xylose. What is the diagnosis?

A) Pancreatic insufficiency
B) Celiac disease -
D xylose is a monosaccharide that can be absorbed in the proximal small intestine, but not in patients
with intestinal mucosal disease, such as Celiac due to atrophy of the intestinal villi in the proximal
small bowel (due to exposure to gluten in wheat)
Q48 Gastrointestinal
A 27-year-old female complains of chronic diarrhea and
steatorrhea. Serum electrolytes are normal, and no microbes are
found in the stool. The D-xylose test is performed, and the
patient's urine is found to have lower than normal levels of D-
xylose. What is the diagnosis?

A) Pancreatic insufficiency - not an absorption problem


B) Celiac disease -
D xylose is a monosaccharide that can be absorbed in the proximal small intestine, but not in patients
with intestinal mucosal disease, such as Celiac due to atrophy of the intestinal villi in the proximal
small bowel (due to exposure to gluten in wheat)
Q49 Gastrointestinal
Strict vegans may not get enough:

A) Vitamin A & folic acid


B) Vitamin D & calcium
Q49 Gastrointestinal
Strict vegans may not get enough:

A) Vitamin A & folic acid


B) Vitamin D & calcium
Q49 Gastrointestinal
Strict vegans may not get enough:

A) Vitamin A & folic acid - in leafy vegetables


B) Vitamin D & calcium - we only get 50% of the vitamin D we
need from sunlight; the other 50% comes from dairy products
and salmon/tuna/other oily fish (esp. in a person who lives in
areas with decreased sunlight); calcium is found in dairy
products
Q50 Gastrointestinal
A 22-year-old female complains of several months of abdominal
discomfort and flatulence after eating. Omeprazole hasn't helped.
She hasn't had fever or vomiting. Her abdomen is nontender, but
somewhat distended. She otherwise appears well. What is the next
step?

A) Rifaximin
B) Amitriptyline
C) Lactose hydrogen breath test
Q50 Gastrointestinal
A 22-year-old female complains of several months of abdominal
discomfort and flatulence after eating. Omeprazole hasn't helped.
She hasn't had fever or vomiting. Her abdomen is nontender, but
somewhat distended. She otherwise appears well. What is the next
step?

A) Rifaximin - for SI bacterial overgrowth (post-surgery)


B) Amitriptyline - TCA, for IBS; rule out LI first
C) Lactose hydrogen breath test - typical of postprandial GI distress in a
healthy patient w/ an unremarkable abdominal exam
Q51 Gastrointestinal
A patient 59-year-old male from Mongolia complains of mid-epigastric
pain (that doesn't change with food intake), nausea, vomiting, and
weight loss. Hepatomegaly is noted on palpation and labs show
microcytic anemia, slightly elevated ALP, ALT, and AST, and
hypoalbuminemia. What is the most likely diagnosis?

A) Pancreatitis
B) Duodenal ulcer
C) Pancreatic cancer
D) Gastric cancer
Q51 Gastrointestinal
A patient 59-year-old male from Mongolia complains of mid-epigastric
pain (that doesn't change with food intake), nausea, vomiting, and
weight loss. Hepatomegaly is noted on palpation and labs show
microcytic anemia, slightly elevated ALP, ALT, and AST, and
hypoalbuminemia. What is the most likely diagnosis?

A) Pancreatitis
B) Duodenal ulcer
C) Pancreatic cancer
D) Gastric cancer
Q51 Gastrointestinal
A patient 59-year-old male from Mongolia complains of mid-epigastric
pain (that doesn't change with food intake), nausea, vomiting, and
weight loss. Hepatomegaly is noted on palpation and labs show
microcytic anemia, slightly elevated ALP, ALT, and AST, and
hypoalbuminemia. What is the most likely diagnosis?

A) Pancreatitis
B) Duodenal ulcer
C) Pancreatic cancer
D) Gastric cancer - greatest risk in Eastern Asia & Eastern Europe (diet);
nausea/vomiting due to stomach obstruction
Q51 Gastrointestinal

Gastric cancer usually presents late in the disease; by that point they are
big enough to cause epigastric pain, weight loss, and
dysphagia/nausea/vomiting due to stomach obstruction.
Tumor vessels often leak blood into the stomach lumen, and so many
patients develop iron deficiency (microcytic anemia). Tumor metastasis
may occur to the liver (hepatomegaly, elevated ALP, ALT, AST, and hypo-
albuminemia).
Q51 Gastrointestinal
A patient 59-year-old male from Mongolia complains of mid-epigastric
pain (that doesn't change with food intake), nausea, vomiting, and
weight loss. Hepatomegaly is noted on palpation and labs show
microcytic anemia, slightly elevated ALP, ALT, and AST, and
hypoalbuminemia. What is the most likely diagnosis?

A) Pancreatitis - pain to back, worse after meals; no hepatomegaly


B) Duodenal ulcer - pain several hours after a meal; no hepatomegaly
C) Pancreatic cancer - epigastric pain, hyperbilirubinemia, jaundice
D) Gastric cancer - greatest risk in Eastern Asia & Eastern Europe (diet);
nausea/vomiting due to stomach obstruction
Q52 Gastrointestinal
A 35-year-old woman with a history of HIV at 29 weeks gestation
presents with elevated transaminases but normal ALP. Her hepatitis C
screen and syphilis were negative at her first prenatal visit. Vitals sign
(including BP) of mom and baby are normal. What is the next step?

A) Urine protein collection


B) Viral hepatitis testing LIVER
ENZY
MES
Q52 Gastrointestinal
A 35-year-old woman with a history of HIV at 29 weeks gestation
presents with elevated transaminases but normal ALP. Her hepatitis C
screen and syphilis were negative at her first prenatal visit. Vitals sign
(including BP) of mom and baby are normal. What is the next step?

A) Urine protein collection


B) Viral hepatitis testing LIVER
ENZY
MES
Q52 Gastrointestinal
A 35-year-old woman with a history of HIV at 29 weeks gestation
presents with elevated transaminases but normal ALP. Her hepatitis C
screen and syphilis were negative at her first prenatal visit. Vitals sign
(including BP) of mom and baby are normal. What is the next step?

A) Urine protein collection - to evaluate preeclampsia;


B) Viral hepatitis testing - elevated transaminases, normal ALP, normal
vital signs, previous HIV "needle use" = likely hepatitis C (disregard initial
prenatal visit, can be contracted later via needle)
Q53 Gastrointestinal
A smoker has weight loss, fatigue, anorexia, and jaundice; ultrasound
reveals that the common bile duct is dilated. This presentation is
consistent with:
A) Wilson disease
B) Pancreatic cancer
Q53 Gastrointestinal
A smoker has weight loss, fatigue, anorexia, and jaundice; ultrasound
reveals that the common bile duct is dilated. This presentation is
consistent with:
A) Wilson disease
B) Pancreatic cancer - the 4th leading cause of cancer death in the US;
tumors in the pancreatic head often compress the pancreatic duct and
common bile duct (jaundice) but there's no pain; tumors in the body tail
don't produce jaundice, but there is abdominal pain; get abdominal
ultrasound; CT scan if ultrasound is unclear or in case of non-jaundice
pancreatic cancer
Q53 Gastrointestinal
FINDINGS IN PANCREATIC ADENOCARCINOMA

-Systemic symptoms (e.g., weight loss) ~90%


-Abdominal/back pain ~80%
-Jaundice ~55%
-Liver metastasis (possible hepatomegaly) ~50%

Remember!! Although CA19-9 is often elevated in pancreatic cancer, it is


NOT used as a screening tool (both because its sensitivity is related to
tumor size and because it is high in other conditions as well, such as
HCC; helpful in monitoring disease progression)
Q54 Gastrointestinal
A 29-year-old diabetic woman at 15 weeks gestation complains of
nausea. She denies vomiting or abdominal pain and her sclerae are
not jaundiced. ALP = 145, AST = 113, ALT = 130. Ultrasound of the
RUQ shows some gallstones (but no thickening of the gallbladder
wall); the common bile duct is normal. What is the diagnosis?
A) Alcoholic hepatitis
B) Acute fatty liver of pregnancy
C) Intrahepatic cholestasis of pregnancy
D) Nonalcoholic fatty liver disease
Q54 Gastrointestinal
A 29-year-old diabetic woman at 15 weeks gestation complains of
nausea. She denies vomiting or abdominal pain and her sclerae are
not jaundiced. ALP = 145, AST = 113, ALT = 130. Ultrasound of the
RUQ shows some gallstones (but no thickening of the gallbladder
wall); the common bile duct is normal. What is the diagnosis?
A) Alcoholic hepatitis
B) Acute fatty liver of pregnancy
C) Intrahepatic cholestasis of pregnancy
D) Nonalcoholic fatty liver disease
Q54 Gastrointestinal
A) Alcoholic hepatitis
-AST would be double ALT
B) Acute fatty liver of pregnancy
-elevated aminotransferase levels, acute RUQ pain and thrombocytopenia (in 3rd trimester)
C) Intrahepatic cholestasis of pregnancy
-elevated aminotransferase levels, elevated total bilirubin levels, generalized pruritis that is worse
on palms and soles
D) Nonalcoholic fatty liver disease
-risk factors: DM, obesity, dyslipidemia; there is an unregulated production of triglycerides in the
liver, which deposit as fatty acids in the liver (leading to steatosis); most patients are
asymptomatic; some have slight RUQ pain; as fats build up, can lead to liver fibrosis and
cirrhosis; Tx: weight loss - reduces the fat deposition in the liver
Q55 Gastrointestinal
The Dr., while reading a CT scan of a 35-year-old woman, incidentally
finds a small mass in the liver that is solitary, well-circumscribed, and
has a central stellate scar. What is the diagnosis?

A) Hepatic adenoma
B) Hepatic hemangioma
C) Focal nodular hyperplasia
Q55 Gastrointestinal
The Dr., while reading a CT scan of a 35-year-old woman, incidentally
finds a small mass in the liver that is solitary, well-circumscribed, and
has a central stellate scar. What is the diagnosis?

A) Hepatic adenoma
B) Hepatic hemangioma
C) Focal nodular hyperplasia
Q55 Gastrointestinal
The Dr., while reading a CT scan of a 35-year-old woman, incidentally
finds a small mass in the liver that is solitary, well-circumscribed, and
has a central stellate scar. What is the diagnosis?

A) Hepatic adenoma - found incidentally, no central scar


B) Hepatic hemangioma - also found incidentally, no central scar
C) Focal nodular hyperplasia - a benign regenerative liver nodule
common in women age 20-50; mostly asymptomatic, usually found
incidentally on imaging; treatment rarely required
Q56 Gastrointestinal
During an episode of benign syncope, a 9-year-old boy fell onto his
abdomen. He complains of postprandial abdominal pain and vomiting
over the next two days. PE shows epigastric tenderness. Bedside
ultrasound shows no free fluid but radiograph shows gastric dilation. What
is the diagnosis?

A) Duodenal hematoma
B) Liver laceration
Q56 Gastrointestinal
During an episode of benign syncope, a 9-year-old boy fell onto his
abdomen. He complains of postprandial abdominal pain and vomiting
over the next two days. PE shows epigastric tenderness. Bedside
ultrasound shows no free fluid but radiograph shows gastric dilation. What
is the diagnosis?

A) Duodenal hematoma
B) Liver laceration
Q56 Gastrointestinal
During an episode of benign syncope, a 9-year-old boy fell onto his
abdomen. He complains of postprandial abdominal pain and vomiting
over the next two days. PE shows epigastric tenderness. Bedside
ultrasound shows no free fluid but radiograph shows gastric dilation. What
is the diagnosis?

A) Duodenal hematoma - seen in BAT; caused by rapid compression of


the duodenum against the spine; delayed presentation as blood builds up
over 1-2 days, until it obstructs the duodenal lumen; patients have signs
of proximal SBO (bilious vomiting, gastric dilation)
B) Liver laceration - also seen in BAT, but RUQ pain, and free fluid
Q57 Gastrointestinal
A 52-year-old male with months of fatigue & joint point comes in due to
progressive abdominal pain. He admits to fevers, weight loss, and
headaches. CT angiogram shows bilateral renal infarcts and several
microaneurysms of abdominal arteries. What is the diagnosis?

A) Thromboangiitis obliterans
B) Polyarteritis nodosa
Q57 Gastrointestinal
A 52-year-old male with months of fatigue & joint point comes in due to
progressive abdominal pain. He admits to fevers, weight loss, and
headaches. CT angiogram shows bilateral renal infarcts and several
microaneurysms of abdominal arteries. What is the diagnosis?

A) Thromboangiitis obliterans
B) Polyarteritis nodosa
Q57 Gastrointestinal
A 52-year-old male with months of fatigue & joint point comes in due to
progressive abdominal pain. He admits to fevers, weight loss, and
headaches. CT angiogram shows bilateral renal infarcts and several
microaneurysms of abdominal arteries. What is the diagnosis?

A) Thromboangiitis obliterans - in smokers; affects the fingers


B) Polyarteritis nodosa - a systemic vasculitis of medium sized arteries;
causes transmural inflammation of arterial walls; fever, joint pain, tissue
ischemia affecting primarily the kidneys (but also the skin, brain, and GI
tract [abdominal pain]), microaneurysms
Q57 Gastrointestinal
Q58 Gastrointestinal
Persistently bloody ascites (on multiple diagnostic paracenteses) would
be most indicative of:

A) Nephrotic syndrome
B) Spontaneous bacterial peritonitis
C) Portal vein thrombosis
D) Hepatocellular carcinoma
Q58 Gastrointestinal
Persistently bloody ascites (on multiple diagnostic paracenteses) would
be most indicative of:

A) Nephrotic syndrome
B) Spontaneous bacterial peritonitis
C) Portal vein thrombosis
D) Hepatocellular carcinoma
Q58 Gastrointestinal
Persistently bloody ascites (on multiple diagnostic paracenteses) would
be most indicative of:

A) Nephrotic syndrome - straw yellow ascites


B) Spontaneous bacterial peritonitis - cloudy ascites
C) Portal vein thrombosis - no ascites (vs. BCS - yellow ascites)
D) Hepatocellular carcinoma - usually blood is due to minor trauma (e.g.,
needle used for paracentesis; but this would have resolved) repeat blood
suggests underlying malignancy - most commonly, HCC, since the tumor
grows and erodes nearby blood vessels)
Q59 Gastrointestinal
A 55-year-old woman develops abdominal distention several days after
surgery. Temp = 98.8F. Bowel sounds are decreased. Imaging shows
diffuse colonic dilation but no obstruction. There is no blood in the stool.
What is the diagnosis?

A) Ischemic colitis
B) E. histolytica
C) Ogilvie syndrome
Q59 Gastrointestinal
A 55-year-old woman develops abdominal distention several days after
surgery. Temp = 98.8F. Bowel sounds are decreased. Imaging shows
diffuse colonic dilation but no obstruction. There is no blood in the stool.
What is the diagnosis?

A) Ischemic colitis
B) E. histolytica
C) Ogilvie syndrome
Q59 Gastrointestinal
A 55-year-old woman develops abdominal distention several days after
surgery. Temp = 98.8F. Bowel sounds are decreased. Imaging shows
diffuse colonic dilation but no obstruction. There is no blood in the stool.
What is the diagnosis?

A) Ischemic colitis - hematochezia; no colonic dilation


B) E. histolytica - hematochezia, fever, sepsis
C) Ogilvie syndrome - "acute colonic pseudoobstruction"; can be caused
by recent infection, surgery, or electrolyte abnormalities; results from
automonic disruption of the colon; tx: bowel rest, colonic decompression;
IV neostigmine if no improvement
Q59 Gastrointestinal
A 55-year-old woman develops abdominal distention several days after
surgery. Temp = 98.8F. Bowel sounds are decreased. Imaging shows
diffuse colonic dilation but no obstruction. There is no blood in the stool.
What is the diagnosis?

A) Ischemic colitis - hematochezia; no colonic dilation


B) E. histolytica - hematochezia, fever, sepsis
C) Ogilvie syndrome - "acute colonic pseudoobstruction"; can be caused
by recent infection, surgery, or electrolyte abnormalities; results from
automonic disruption of the colon; tx: bowel rest, colonic decompression;
IV neostigmine if no improvement
Q59 Gastrointestinal

THE OGRE WILL GIVE ME "PSEUDO-


OBSTRUCTION"
Q60 Gastrointestinal
A 40-year-old woman presents with months of jaundice and weight loss.
Aminotransferases are slightly elevated and ALP = 890. What is the most
likely diagnosis?

A) Acute choledocholithiasis
B) Autoimmune hepatitis
C) Malignant biliary obstruction
Q60 Gastrointestinal
A 40-year-old woman presents with months of jaundice and weight loss.
Aminotransferases are slightly elevated and ALP = 890. What is the most
likely diagnosis?

A) Acute choledocholithiasis
B) Autoimmune hepatitis
C) Malignant biliary obstruction
Q60 Gastrointestinal
A 40-year-old woman presents with months of jaundice and weight loss.
Aminotransferases are slightly elevated and ALP = 890. What is the most
likely diagnosis?

A) Acute choledocholithiasis - acute RUQ pain


B) Autoimmune hepatitis - ALP not significantly elevated
C) Malignant biliary obstruction - elevated ALP out of proportion to
transaminases suggests intrahepatic cholestasis or biliary obstruction; this
plus weight loss hints to malignancy (e.g., pancreatic adenocacinoma or
cholangiocarcinoma)
Q61 Gastrointestinal
A little girl takes an over-the-counter medication for fever, which
resolved, but then soon develops nausea, vomiting, hepatomegaly,
and cerebral edema. What is the mechanism?

A) Excess urea
B) Mitochondrial dysfunction
Q61 Gastrointestinal
A little girl takes an over-the-counter medication for fever, which
resolved, but then soon develops nausea, vomiting, hepatomegaly,
and cerebral edema. What is the mechanism?

A) Excess urea
B) Mitochondrial dysfunction
Q61 Gastrointestinal
A) Excess urea - uremic encephalopathy (no c. edema)
B) Mitochondrial dysfunction - Reye syndrome (a type of toxic-
metabolic encephalopathy - brain dysfunction in the absence of
primary CNS disease); aspirin causes mitochondrial dysfunction,
leading to impaired fatty acid metabolism and acute hepatic
steatosis (hepatomegaly), ammonia accumulates (cerebral
edema and encephalopathy - can lead to seizures and death); tx:
supportive - only give aspirin to kids in Kawasaki and
rheumatologic diseases (e.g., juvenile idiopathic arthritis)
Q62 Gastrointestinal
A 7-month old baby (with a history of abdominal surgery soon after
birth) presents with chronic symptoms of episodic bilious vomiting,
abdominal pain, irritability and inconsolability. Bowel sounds are
normal. Stool is negative for blood. What is the most likely
diagnosis?

A) Pyloric stenosis
B) Food protein induced enterocolitis
C) Intestinal malrotation
Q62 Gastrointestinal
A 7-month old baby (with a history of abdominal surgery soon after
birth) presents with chronic symptoms of episodic bilious vomiting,
abdominal pain, irritability and inconsolability. Bowel sounds are
normal. Stool is negative for blood. What is the most likely
diagnosis?

A) Pyloric stenosis
B) Food protein induced enterocolitis
C) Intestinal malrotation (volvulus)
Q62 Gastrointestinal
A) Pyloric stenosis - presents at age 3-5 weeks with NON-bilious
vomiting after EVERY feed
B) Food protein induced enterocolitis - a non-IgE mediated food
allergy that presents in infancy with NON-bilious vomiting after
ingestion of a trigger food (e.g., milk)
C) Intestinal malrotation - a common cause of bilious emesis in
infants, especially those with other congenital anomalies (eg,
omphalocele) or adhesions. Chronic, episodic symptoms can occur
with intermittent volvulus (acute volvulus is a life-threatening
emergency); dx: upper GI series. tx: surgery
Q62 Gastrointestinal
A) Pyloric stenosis - presents at age 3-5 weeks with NON-
bilious vomiting after EVERY feed
B) Food protein induced enterocolitis - a non-IgE mediated
food allergy that presents in infancy with NON-bilious
vomiting after ingestion of a trigger food (e.g., milk)
C) Intestinal malrotation - a common cause of bilious
emesis in infants, especially those with other congenital
anomalies (eg, omphalocele) or adhesions. Chronic,
episodic symptoms can occur with intermittent volvulus
(acute volvulus is a life-threatening emergency); dx: upper
GI series. tx: surgery
Q62 Gastrointestinal
If you see this x-ray of a
hemodynamically unstable patient (i.e.,
hypotension, tachycardia), OR in a
patient with a rigid, tender abdomen, the
answer is

EXPLORATORY LAPAROTOMY
IMMEDIATELY!!!

It can rapidly lead to ischemia, necrosis,


and shock! Don't wait for Upper GI series
-
that's reserved for hemodynamically
stSTABLE patients.
Q63 Gastrointestinal
A 39-year-old woman ingested acid to commit suicide 3 months
ago (after finding out that she was diagnosed DM). Now, she
develops early satiety, nausea, nonbilous vomiting, and weight
loss. What is the cause?

A) Diabetic gastroparesis
B) Pyloric stricture
Q63 Gastrointestinal
A 39-year-old woman ingested acid to commit suicide 3 months
ago (after finding out that she was diagnosed DM). Now, she
develops early satiety, nausea, nonbilous vomiting, and weight
loss. What is the cause?

A) Diabetic gastroparesis
B) Pyloric stricture
Q63 Gastrointestinal
A) Diabetic gastroparesis - DM > 10 years
B) Pyloric stricture - the signs and symptoms
Pyloric stricture
are consistent with a gastric outlet
obstruction, leading to postprandial pain and
vomiting; causes of GOO include gastric
malignancy, Crohn disease, and strictures
(e.g., fibrosis that builds up weeks after acid
ingestion; dx: upper endoscopy; tx: surgery)
Q64 Gastrointestinal
A patient with cirrhosis that develops confusion and asterixis
should be treated with:

A) Dantrolene
B) Lactulose
C) Chlordiazepoxide
Q64 Gastrointestinal
A patient with cirrhosis that develops confusion and asterixis
should be treated with:

A) Dantrolene
B) Lactulose
C) Chlordiazepoxide
Q64 Gastrointestinal
A patient with cirrhosis that develops confusion and asterixis should be
treated with:

A) Dantrolene - used in NPS and MH (high fever, muscle rigidity)


B) Lactulose - to lower ammonia in hepatic encephalopathy (due to
ammonia neurotoxicity from impaired liver function); colonic bacteria
break down lactulose to short chain fatty acids, which acidifies the
colon, stimulating conversion of absorbable ammonium to the
nonabsorbable form
C) Chlordiazepoxide - LA benzodiazepine for alcohol withdrawal
Q65 Gastrointestinal
A 40-year-old woman complains of epigastric pain and diarrhea.
PE shows abdominal tender; endoscopy reveals gastric folds and
several duodenal ulcers. What is the next step?

A) H. pylori triple therapy


B) Serum gastrin level
Q65 Gastrointestinal
A 40-year-old woman complains of epigastric pain and diarrhea.
PE shows abdominal tender; endoscopy reveals gastric folds and
several duodenal ulcers. What is the next step?

A) H. pylori triple therapy


B) Serum gastrin level
Q65 Gastrointestinal
A 40-year-old woman complains of epigastric pain and diarrhea.
PE shows abdominal tender; endoscopy reveals gastric folds and
several duodenal ulcers. What is the next step?

A) H. pylori triple therapy - Triple therapy treats H pylori infection,


which can cause gastric/duodenal ulcers. However, diarrhea and
multiple ulcers in different sites are more suggestive of gastrinoma.
B) Serum gastrin level - first step in diagnosis of ZES
Q66 Gastrointestinal
A woman in her third trimester has epigastric pain radiating to the
back, and lipase = 530 U/L. She doesn't drink and gallstones are
excluded. She has a history of triglyceridemia. What is the next
step?

A) antibiotics
B) CT scan
C) lipid panel
Q66 Gastrointestinal
A woman in her third trimester has epigastric pain radiating to the
back, and lipase = 530 U/L. She doesn't drink and gallstones are
excluded. She has a history of triglyceridemia. What is the next
step?

A) antibiotics
B) CT scan
C) lipid panel
Q66 Gastrointestinal
A) antibiotics - no benefit in acute pancreatitis (except in infected
pancreatic necrosis and extrapancreatic infection that may develop)
B) CT scan - we already have epigastric pain and lipase levels
C) lipid panel - this woman likely his trigleride induced pancreatitis;
fats increase in pregnancy by 2-4 times, especially in the third
trimester; a triglyc. level > 1000 mg/dL required for diagnosis of
triglyceride induced pancreatitis; besides IV fluids and pain control
(like all cases of acute pancreatitis), managment may include insulin
or apheresis.
Q67 Gastrointestinal
What is the treatment for an asymptomatic biliary
cyst?

A) reassurance
B) surgery
Q67 Gastrointestinal
What is the treatment for an asymptomatic biliary
cyst?

A) reassurance
B) surgery
Q67 Gastrointestinal
What is the treatment for an asymptomatic biliary
cyst?

A) reassurance
B) surgery - due to concern for malignancy, such as
cholangiocarcinoma or gallbladder cancer; (may
present with RUQ pain, mass, and/or jaundice;
minority of patients are asymptomatic)
Q68 Gastrointestinal
A 45-year-old obese woman develops mild
hepatomegaly with AST = 84 U/L and ALT = 94 U/L.
She has diabetes and has been taking metformin and
atorvastatin for years. What is the diagnosis?

A) Statin liver injury


B) Autoimmune hepatitis
C) NAFLD
Q68 Gastrointestinal
A 45-year-old obese woman develops mild
hepatomegaly with AST = 84 U/L and ALT = 94 U/L.
She has diabetes and has been taking metformin and
atorvastatin for years. What is the diagnosis?

A) Statin liver injury


B) Autoimmune hepatitis
C) NAFLD
Q68 Gastrointestinal
A 45-year-old obese woman develops mild
hepatomegaly with AST = 84 U/L and ALT = 94 U/L.
She has diabetes and has been taking metformin and
atorvastatin for years. What is the diagnosis?
A) Statin liver injury - liver injury w/ in 3 months of therapy
B) Autoimmune hepatitis - much higher AST, ALT; often ass./w
arthralgia, anorexia, and fatigue
C) NAFLD - (obesity) hepatomegaly, elevated ALT & AST (treat with
weight loss and diet)
Q69 Gastrointestinal
A lady presents with severe epigastric pain, nausea,
and vomiting 24 hours after undergoing ERCP. What
is the next step?

A) Serum lipase
B) CT scan of the abdomen
Q69 Gastrointestinal
A lady presents with severe epigastric pain, nausea,
and vomiting 24 hours after undergoing ERCP. What
is the next step?

A) Serum lipase
B) CT scan of the abdomen
Q69 Gastrointestinal
A lady presents with severe epigastric pain, nausea,
and vomiting 24 hours after undergoing ERCP. What
is the next step?

A) Serum lipase - ERCP can lead to acute


pancreatitis
B) CT scan of the abdomen- amylase & lipase go up
right away; CT scan may be normal for 48 hours
Q70 Gastrointestinal
A 23-year-old man has involuntary hand shaking,
jerky movements of his extremities, and
hepatomegaly. AST = 276 U/L and ALT = 130 U/L. He
doesn't drink. What is the diagnosis?

A) Hemochromatosis
B) Wilson disease
Q70 Gastrointestinal
A 23-year-old man has involuntary hand shaking,
jerky movements of his extremities, and
hepatomegaly. AST = 276 U/L and ALT = 130 U/L. He
doesn't drink. What is the diagnosis?

A) Hemochromatosis
B) Wilson disease
Q70 Gastrointestinal
Wilson Disease
-autosomal recessive disorder that results in defective copper transport and
subsequent accumulation and deposition of copper in the liver and brain; usually in
patients <30
-patients present with hepatitis/cirrhosis, neurologic dysfunction (ataxia, tremor), &
psychiatric abnormalities (psychosis, anxiety, depression)
-PE: may show Kayser-Fleischer rings (green to brown copper deposits), jaundice,
hepatomegaly, asterixis, jerky movements, and rigidity
-Dx: slit lamp exam (best initial), increased urinary copper excretion after giving
penicillamine (most accurate)
-Tx: penicillamine or trientine (copper chelators), dietary copper restriction, and
zinc (to increase fecal excretion)
Q71 Gastrointestinal
Self induced vomiting + chest/back pain + plueral
effusion = ?
Q71 Gastrointestinal
Self induced vomiting + chest/back pain + plueral
effusion =
BOERHAAVE SYNDROME!
esophageal perforation
(w/ pleural effusion)
Q71 Gastrointestinal
Self induced vomiting + chest/back pain + plueral effusion
=
BOERHAAVE SYNDROME!
esophageal perforation
(w/ pleural effusion)
-Can occur with repeated vomiting; patients usually have severe chest and/or
back pain; often have a systemic inflammatory response (fever, tachycardia);
leakage to pleural space -> pleural effusion
-give IV broad spectrum antibiotics and PPI's; emergency surgery
Q72 Gastrointestinal
4-year-old girl with short stature has dysphagia to
solids, vomiting, and recurrent food impactions. What
is the diagnosis?

A) Vascular ring
B) Esophageal ganglion degeneration
Q72 Gastrointestinal
4-year-old girl with short stature has dysphagia to
solids, vomiting, and recurrent food impactions. What
is the diagnosis?

A) Vascular ring
B) Esophageal ganglion degeneration
Q72 Gastrointestinal

A) Vascular ring - can compress trachea (biphasic


stridor) or the esophagus (severe dysphagia, more
pronounced at age 6 months when baby starts solids;
tx: surgery)
B) Esophageal ganglion degeneration - achalasia;
dysphagia to both solids and liquids
Q73 Gastrointestinal
All patients with cirrhosis should undergo which
test?

A) Endoscopy
B) CT scan
Q73 Gastrointestinal
All patients with cirrhosis should undergo which
test?

A) Endoscopy
B) CT scan
Q73 Gastrointestinal
All patients with cirrhosis should undergo which
test?

A) Endoscopy -
all patients with cirrhosis should undergo screening endoscopy to
exclude varices, determine the risk of variceal hemorrhage, and provide
strategies for prevention of hemorrhage.
B) CT scan - unnecessary
Q74 Gastrointestinal
If EGD reveals some varices, what is the next
step?
Q74 Gastrointestinal
If EGD reveals some varices, what is the next
step?

Nonselective BB's (propranolol & nadolol) OR


Endoscopic variceal ligation

(patient choice)
Q75 Gastrointestinal
A pregnant woman in her third trimester has mildly
elevated aminotransferases, hypoglycemia, and
thrombocytopenia. She is normotensive. What is
the treatment?

A) Magnesium
B) Delivery
Q75 Gastrointestinal
A pregnant woman in her third trimester has mildly
elevated aminotransferases, hypoglycemia, and
thrombocytopenia. She is normotensive. What is
the treatment?

A) Magnesium
B) Delivery
Q75 Gastrointestinal
A pregnant woman in her third trimester has mildly
elevated aminotransferases, hypoglycemia, and
thrombocytopenia. She is normotensive. What is
the treatment?

A) Magnesium - preeclampsia
B) Delivery: for acute fatty liver of pregnancy
Q76 Gastrointestinal
A patient with Tylenol toxicity has rising serum
bilirubin levels and PT level. What is the next step?

A) Vitamin K
B) Send for liver transplant
Q76 Gastrointestinal
A patient with Tylenol toxicity has rising serum
bilirubin levels and PT level. What is the next step?

A) Vitamin K
B) Send for liver transplant
Q77 Gastrointestinal
Spider angiomas and palmar erythema in cirrhosis
are due to:

A) ammonia
B) estrogen
Q77 Gastrointestinal
Spider angiomas and palmar erythema in cirrhosis
are due to:

A) ammonia
B) estrogen
Q77 Gastrointestinal
Spider angiomas and palmar erythema in cirrhosis
are due to:

A) ammonia
B) estrogen
impaired hepatic metabolism of circulating estrogens leads to hyperestrinism
(e.g., gynecomastia, testicular atrophy); circulating estrogens affect vascular
dilation; this explains spider angiomas (dilated arteriole surrounded by small
radiating vessels) and palmar erythema
Q78 Gastrointestinal
A relatively asymptomatic man has hepatosplenomegaly,
elevated LFT's, hypercalcemia, & hilar lymphadenopathy.
What is the diagnosis?

A) Hemochromatosis
B) Sarcoidosis
Q78 Gastrointestinal
A relatively asymptomatic man has hepatosplenomegaly,
elevated LFT's, hypercalcemia, & hilar lymphadenopathy.
What is the diagnosis?

A) Hemochromatosis
B) Sarcoidosis
Q78 Gastrointestinal
A relatively asymptomatic man has hepatosplenomegaly,
elevated LFT's, hypercalcemia, & hilar lymphadenopathy.
What is the diagnosis?

A) Hemochromatosis - lung involvement unlikely


B) Sarcoidosis
a multisystemic inflammatory disease involving noncaseating granulomas. Asymptomatic
liver involvement (LFT's, hepatomegaly) is common (~65%); also can affect the spleen
(splenomegaly); lung involvement is extremely common (>90%); hypercalcemia results
from vitamin D conversion by macrophages in granulomas
Q79 Gastrointestinal
What is the proper management for a button battery
lodged in the esophagus (e.g., of a baby)?

A) Bronchoscopy
B) Endoscopy
Q79 Gastrointestinal
What is the proper management for a button battery
lodged in the esophagus (e.g., of a baby)?

A) Bronchoscopy
B) Endoscopy
Q79 Gastrointestinal
What is the proper management for a button battery
lodged in the esophagus (e.g., of a baby)?

A) Bronchoscopy - for trachea (e.g., an aspirated object),


not esophagus
B) Endoscopy - immediate endoscopic removal
Q79 Gastrointestinal
What is the proper management for a button battery
lodged in the esophagus (e.g., of a baby)?

A) Bronchoscopy - for trachea (e.g., an aspirated object),


not esophagus
B) Endoscopy - immediate endoscopic removal
(the same would be true for any sharp object [e.g., fishbone] found in
the esophagus, stomach, or proximal* duodenum; OR, in case of 2
magnets, due to risk of bowel entrapment*)
Q80 Gastrointestinal
A 2-year-old with impaired adaptation to darkness,
photobia, and dry scaly skin likely has which vitamin
deficiency?

A) Vitamin A
B) Vitamin B2
C) Vitamin C
Q80 Gastrointestinal
A 2-year-old with impaired adaptation to darkness,
photobia, and dry scaly skin likely has which vitamin
deficiency?

A) Vitamin A
B) Vitamin B2
C) Vitamin C
Q80 Gastrointestinal
A 2-year-old with impaired adaptation to darkness,
photobia, and dry scaly skin likely has which vitamin
deficiency?

A) Vitamin A - photobia, dry skin, Bitot spots


B) Vitamin B2 - sore throat, cheilitis, glossitis
C) Vitamin C - ecchymoses, petechiae, bleeding gums
Q81 Gastrointestinal
A 70-year-old woman with atherosclerosis presents with
crampy postprandial epigastric pain, food aversion, and weight
loss. What is the underlying mechanism of the most likely
diagnosis?

A) Selective hypersensitization of visceral afferent nerves


B) Atherosclerosis of the mesenteric arteries
Q81 Gastrointestinal
A 70-year-old woman with atherosclerosis presents with
crampy postprandial epigastric pain, food aversion, and weight
loss. What is the underlying mechanism of the most likely
diagnosis?

A) Selective hypersensitization of visceral afferent nerves


B) Atherosclerosis of the mesenteric arteries
Q81 Gastrointestinal
A 70-year-old woman with atherosclerosis presents with
crampy postprandial epigastric pain, food aversion, and weight
loss. What is the underlying mechanism of the most likely
diagnosis?

A) Selective hypersensitization of visceral afferent nerves- IBS


B) Atherosclerosis of the mesenteric arteries - chronic mesenteric
ischemia (may also see nausea, diarrhea, abdominal bruit in ~50%
of patients)
Q82 Gastrointestinal
An elderly lady presents with conjunctival pallor. Which
medication may have caused this?

A) Aspirin & naproxen


B) Levothyroxine
Q82 Gastrointestinal
An elderly lady presents with conjunctival pallor. Which
medication may have caused this?

A) Aspirin & naproxen


B) Levothyroxine
Q82 Gastrointestinal
An elderly lady presents with conjunctival pallor. Which
medication may have caused this?

A) Aspirin & naproxen - conjunctival pallor is nonspecific, but


suggests an underlying anemia; concurrent NSAID and aspirin
can cause gastritis and/or gastric ulcers leading to GI blood
loss and depletion of iron stores
B) Levothyroxine
Q83 Gastrointestinal
A 65-year-old man has early satiety, nausea, abdominal
distention, and intractable vomiting. He also has hypokalemia
and new onset hyperglycemia. What is the most likely
diagnosis?

A) Gastric bezoar
B) Pancreatic adenocarcinoma
Q83 Gastrointestinal
A 65-year-old man has early satiety, nausea, abdominal
distention, and intractable vomiting. He also has hypokalemia
and new onset hyperglycemia. What is the most likely
diagnosis?

A) Gastric bezoar
B) Pancreatic adenocarcinoma
Q83 Gastrointestinal
A 65-year-old man has early satiety, nausea, abdominal
distention, and intractable vomiting. He also has hypokalemia
and new onset hyperglycemia. What is the most likely
diagnosis?

A) Gastric bezoar - indigestible materials in stomach; very rare


B) Pancreatic adenocarcinoma
the patient shows signs of GOO (mechanical obstruction of the stomach or duodenum;
hypokalemia due to vomiting); 80% are caused by malignancy (most common pancreatic
adenocarcinoma; this explains the hyperglycemia - due to islet cell destruction as tumor invades
the pancreas)
Q84 Gastrointestinal
A pregnant woman is found to have elevated AFP on 2nd
trimester screening. Ultrasound shows a fetus with thickened
intestinal loops floating free in the amniotic fluid. What is the
diagnosis?

A) Gastroschisis
B) Prune belly
Q84 Gastrointestinal
A pregnant woman is found to have elevated AFP on 2nd
trimester screening. Ultrasound shows a fetus with thickened
intestinal loops floating free in the amniotic fluid. What is the
diagnosis?

A) Gastroschisis
B) Prune belly
Q84 Gastrointestinal
A pregnant woman is found to have elevated AFP on 2nd
trimester screening. Ultrasound shows a fetus with thickened
intestinal loops floating free in the amniotic fluid. What is the
diagnosis?

A) Gastroschisis - a full-thickness abdominal wall defect;


herniation of uncovered bowel allows AFP to pass through the
exposed bowel into the amniotic fluid; tx: surgery
B) Prune belly - triad includes omphacele (looks like prune)
Q84 Gastrointestinal
Q85 Gastrointestinal
Distant infection (e.g., endocarditis) + high fever + tender
splenomegaly =

A) Non-Hodgkin lymphoma
B) Splenic abscess
Q85 Gastrointestinal
Distant infection (e.g., endocarditis) + high fever + tender
splenomegaly =

A) Non-Hodgkin lymphoma
B) Splenic abscess
Q85 Gastrointestinal
Distant infection (e.g., endocarditis) + high fever + tender
splenomegaly =

A) Non-Hodgkin lymphoma - high fevers uncommon


B) Splenic abscess - a rare, potentially fatal complication of
bacteremia from a distant infection; risk greatest in
immunocompromised from HIV, malignancy, or DM; patients
usually have persistent fever and LUQ pain; tx: antibiotics and
splenectomy
Q86 Gastrointestinal
A baby has these finding. What screening test should be done?

A) Abdominal ultrasound
B) Echocardiogram
Q86 Gastrointestinal
A baby has these finding. What screening test should be done?

A) Abdominal ultrasound
B) Echocardiogram
Q86 Gastrointestinal
A baby has these finding. What screening test should be done?

A) Abdominal ultrasound -
need to screen for
Wilms tumor
B) Echocardiogram
Q87 Gastrointestinal
A patient has dysphagia solids but not liquids. He feels like food
gets "stuck in his chest". What is the next step?

A) endoscopy
B) manometry
Q87 Gastrointestinal
A patient has dysphagia solids but not liquids. He feels like food
gets "stuck in his chest". What is the next step?

A) endoscopy
B) manometry
Q87 Gastrointestinal
A patient has dysphagia solids but not liquids. He feels like food
gets "stuck in his chest". What is the next step?

A) endoscopy - first step in assessing suspected mechanical


obstruction (e.g., malignancy)
B) manometry - achalasia involves dysphagia to both solids and
liquids; manometry done around the same time as upper GI
endoscopy
Q88 Gastrointestinal
A pregnant lady presents with right sided abdominal pain (with
rebound and guarding), fever, and mild leukocytosis several
days after a C section. What is the diagnosis?

A) Appendicitis
B) Septic pelvic thrombophlebitis
C) Endometritis
Q88 Gastrointestinal
A pregnant lady presents with right sided abdominal pain (with
rebound and guarding), fever, and mild leukocytosis several
days after a C section. What is the diagnosis?

A) Appendicitis
B) Septic pelvic thrombophlebitis
C) Endometritis
Q88 Gastrointestinal
A pregnant lady presents with right sided abdominal pain (with
rebound and guarding), fever, and mild leukocytosis several
days after a C section. What is the diagnosis?

A) Appendicitis
B) Septic pelvic thrombophlebitis
C) Endometritis
Pregnant patients may have an atypical
appendicitis presentation of pain in the right
mid-to-upper quadrant or right flank.
Q88 Gastrointestinal
A) Appendicitis - periumbilical pain masked post C section; as ischemia
develops, patient develops systemic symptoms (fever, leukocytosis) and
localized signs of inflammation (rebound and guarding due to peritoneal
involvement)
B) Septic pelvic thrombophlebitis - rare; associated with endometritis;
relapsing-remitting fevers; no rebound or guarding because infection is
contained in the retroperitoneum
C) Endometritis - fever and lower abdominal pain; also uterine tenderness,
purulent lochia, and heavy vaginal bleeding (not seen in this patient); no
rebound or guarding because infection is contained in the uterine cavity
Q89 Gastrointestinal
A toddler with constipation but normal growth should
be treated with:

A) polyethylene glycol
B) loperamide
Q89 Gastrointestinal
A toddler with constipation but normal growth should
be treated with:

A) polyethylene glycol
B) loperamide
Q89 Gastrointestinal
A toddler with constipation but normal growth should
be treated with:

A) polyethylene glycol - i.e., Miralax; constipation is


normal in toddlers (e.g., change to solid food, change
to milk, beginning toilet training, starting school which
causes stool withholding)
B) loperamide - an anti-diarrheal agent
Q90 Gastrointestinal
A 26-year-old woman has episodic epigastric pain. The pain, which
wakes her up at night, is relieved by some food and water. She has no
signficant medical history and takes no meds. She admits to drinking
alcohol and smoking pot. After confirmation of the diagnosis, which is the
next best step to provide relief for her symptoms?

A) Stop alcohol
B) Stop pot
C) Omeprazole
D) Antibiotics and pantoprazole
Q90 Gastrointestinal
A 26-year-old woman has episodic epigastric pain. The pain, which
wakes her up at night, is relieved by some food and water. She has no
signficant medical history and takes no meds. She admits to drinking
alcohol and smoking pot. After confirmation of the diagnosis, which is the
next best step to provide relief for her symptoms?

A) Stop alcohol
B) Stop pot
C) Omeprazole
D) Antibiotics and pantoprazole
Q90 Gastrointestinal

A) Stop alcohol - light alcohol intake doesn't affect DU's


B) Stop pot - irrelevant to DU
C) Omeprazole - not enough
D) Antibiotics and pantoprazole - duodenal ulcer pain generally
decreases after eating (due to alkaline fluid secretion into the duodenum);
majority of duodenal ulcers due to H pylori or NSAIDs (but she doesn't
take meds); so we confirm the diagnosis of H pylori with urea breath test
or endoscopic biopsy; management involves antibiotics (amox + clarith)
and a PPI (omeprazole or pantoprazole)
Q91 Gastrointestinal
An elderly patient has vague left lower abdominal discomfort,
anorexia, and constipation; he experiences a sudden severe
episode of abdominal pain, followed by fever and peritonitis. Free
air is seen on abdominal imaging. What happened?

A) Embolic occlusion of a mesenteric artery


B) Diverticular perforation
Q91 Gastrointestinal
An elderly patient has vague left lower abdominal discomfort,
anorexia, and constipation; he experiences a sudden severe
episode of abdominal pain, followed by fever and peritonitis. Free
air is seen on abdominal imaging. What happened?

A) Embolic occlusion of a mesenteric artery


B) Diverticular perforation
Q91 Gastrointestinal
An elderly patient has vague left lower abdominal discomfort,
anorexia, and constipation; he experiences a sudden severe
episode of abdominal pain, followed by fever and peritonitis. Free
air is seen on abdominal imaging. What happened?

A) Embolic occlusion of a mesenteric artery


B) Diverticular perforation - classic presentation!
Q92 Gastrointestinal
Which diagnostic markers are most appropriate to
diagnose acute hepatitis B infection?

A) HbcAg
B) HbeAg
C) HbsAg and IgM anti-HBc
Q92 Gastrointestinal
Which diagnostic markers are most appropriate to
diagnose acute hepatitis B infection?

A) HbcAg
B) HbeAg
C) HbsAg and IgM anti-HBc
Q92 Gastrointestinal
Which diagnostic markers are most appropriate to
diagnose acute hepatitis B infection?

C) HbsAg and IgM anti-HBc - the first serologic marker


to appear in the serum with acute HBV is HbsAg (4-8
weeks after infection); IgM soon after, which is when
clinical symptoms occur, and elevations in
aminotransferases
Q93 Gastrointestinal
A 7-week-old healthy baby strains when pooping. The
stool has normal consistency. Next step?

A) barium enema
B) sweat chloride test
C) reassurance
Q93 Gastrointestinal
A 7-week-old healthy baby strains when pooping. The
stool has normal consistency. Next step?

A) barium enema
B) sweat chloride test
C) reassurance
Q93 Gastrointestinal
A 7-week-old healthy baby strains when pooping. The
stool has normal consistency. Next step?

A) barium enema
B) sweat chloride test
C) reassurance - infant dyschezia is a common,
benign, self-resolving functional disorder in which the
baby cries and strains before pooping
Q94 Gastrointestinal
Sepsis (e.g., fever, hypotension, confusion) 10 days
after admission for acute pancreatitis warrants:

A) CT scan of the abdomen


B) lumbar puncture
Q94 Gastrointestinal
Sepsis (e.g., fever, hypotension, confusion) 10 days
after admission for acute pancreatitis warrants:

A) CT scan of the abdomen


B) lumbar puncture
Q94 Gastrointestinal
Sepsis (e.g., fever, hypotension, confusion) 10 days
after admission for acute pancreatitis warrants:

A) CT scan of the abdomen - to assess for infected


pancreatic necrosis; tx: antibiotics & debridement
B) lumbar puncture - meningitis (unlikely in the
absence of nuchal rigidity or severe headache)
Q95 Gastrointestinal
A patient who underwent Roux-en-Y gastric bypass
several months ago develops nausea, postprandial
vomiting, GERD and dysphagia to solids and liquids.
What is the next step?

A) EGD
B) Gastric emptying scan
Q95 Gastrointestinal
A patient who underwent Roux-en-Y gastric bypass
several months ago develops nausea, postprandial
vomiting, GERD and dysphagia to solids and liquids.
What is the next step?

A) EGD
B) Gastric emptying scan
Q95 Gastrointestinal

A) EGD - Roux-en-Y has several complications


including stomal stenosis (in first year; involves
narrowing of the GJ anastomosis); dx: with EGD,
during which balloon dilation can be performed to open
the narrowed gastric pouch outflow
B) Gastric emptying scan
Q96 Gastrointestinal
A patient has fever, fatigue, weight loss, and severely
elevated aminotransferases (>2000). What is the
diagnosis?

A) Acute cholangitis
B) Hepatitis A infection
Q96 Gastrointestinal
A patient has fever, fatigue, weight loss, and severely
elevated aminotransferases (>2000). What is the
diagnosis?

A) Acute cholangitis
B) Hepatitis A infection
Q96 Gastrointestinal
A) Acute cholangitis - very sick, elevated ALP, but ALT &
AST not significantly elevated
B) Hepatitis A infection - abrupt onset fever, fatigue,
anorexia, severely elevated aminotransferases; usually
followed by jaundice and pruritis within 2 weeks (~55% of
time); (fecal-oral transmission); dx: confirmed by anti-HAV
IgM; tx: supportive care, post-exposure prophylaxis
(vaccine, HA Ig) to close contacts
Q97 Gastrointestinal
A child develops rapid encephalopathy and liver failure
(hepatomegaly, aminotransferases), as well as
elevated ammonia. No jaundice is noted. What should
be assessed for to obtain a diagnosis?

A) Immunization status
B) Medication history
Q97 Gastrointestinal
A child develops rapid encephalopathy and liver failure
(hepatomegaly, aminotransferases), as well as
elevated ammonia. No jaundice is noted. What should
be assessed for to obtain a diagnosis?

A) Immunization status
B) Medication history
Q97 Gastrointestinal
A child develops rapid encephalopathy and liver failure
(hepatomegaly, aminotransferases), as well as
elevated ammonia. [Reye syndrome! Seen in young
patients on aspirin for a viral infection. (Mitochondrial
dysfunction → hepatic dysfunction →ammonia
accumulates → astrocyte edema)]
A) Immunization status
B) Medication history
Q98 Gastrointestinal
A patient presents with melena, RUQ pain, jaundice,
anemia, and hyperbilirubinemia after a recent liver
biopsy. Hb = 8.7 g/d/L. What is the most likely
diagnosis?

A) Acute cholecystitis
B) Haemobilia
Q98 Gastrointestinal
A patient presents with melena, RUQ pain, jaundice,
anemia, and hyperbilirubinemia after a recent liver
biopsy. Hb = 8.7 g/d/L. What is the most likely
diagnosis?

A) Acute cholecystitis
B) Haemobilia
Q98 Gastrointestinal
A) Acute cholecystitis
RUQ pain and leukocytosis but not hyper-bilirubinemia
or jaundice or melena or anemia
B) Haemobilia
involves bleeding into the biliary tract (RUQ pain,
jaundice, anemia, hyperbilirubinemia); esp. several
days after a procedure like liver biopsy or
cholecystectomy, or due to tumor or trauma
Q99 Gastrointestinal
All breastfed infants should receive vitamin D. T/F.

A) True
B) False
Q99 Gastrointestinal
All breastfed infants should receive vitamin D. T/F.

A) True
B) False
Q99 Gastrointestinal
All breastfed infants should receive vitamin D. T/F.

A) True - breast milk has inadequate vitamin D


B) False
Q100 Gastrointestinal
A patient with a history of cirrhosis and ascites
develops fever, lethargy, and mental status changes.
What should be done next?

A) Barium enema
B) Laparoscopy
C) Paracentesis
Q100 Gastrointestinal
A patient with a history of cirrhosis and ascites
develops fever, lethargy, and mental status changes.
What should be done next?

A) Barium enema
B) Laparoscopy
C) Paracentesis
Q100 Gastrointestinal
A patient with a history of cirrhosis and ascites
develops fever, lethargy, and mental status changes.
What should be done next?

A) Barium enema - used to assess colon


B) Laparoscopy - not necessary for diagnosis
C) Paracentesis - to assess for suspected SBP
Q100 Gastrointestinal
C) Paracentesis - to assess for suspected SBP
Paracentesis is used to diagnose SBP (diagnostic if positive
ascites fluid culture and neutrophil count > 250/mm3); should
be done before antibiotic therapy is initiated as therapy often
results in negative ascites cultures. Enteric organisms such as
E. coli and Klebsiella are the most commonly cultured
organisms followed by the strep species; empiric therapy
usually includes a third-generation cephalosporin.
Q101 Gastrointestinal
A woman on chemotherapy develops voluminous watery
brown diarrhea that persists despite not eating. Stool tests are
negative for fecal occult blood and C diff. Leukocytes =
5,500/mm3. What is the best next step?

A) Cholestyramine
B) Loperamide
C) Sulfasalazine
D) Prednisone
Q101 Gastrointestinal
A woman on chemotherapy develops voluminous watery
brown diarrhea that persists despite not eating. Stool tests are
negative for fecal occult blood and C diff. Leukocytes =
5,500/mm3. What is the best next step?

A) Cholestyramine
B) Loperamide
C) Sulfasalazine
D) Prednisone
Q101 Gastrointestinal
A) Cholestyramine - for chronic diarrhea related to bile acid
malabsorption, usually seen after surgical resection of the
small bowel or gallbladder
B) Loperamide - antidiarrheal (plus oral hydration); diarrhea is
common with chemotherapy (e.g., fluorouracil) due to direct
cytotoxic effects on intestinal mucosa
C) Sulfasalazine - for IBD (hematochezia, anemia,
leukocytosis)
D) Prednisone - also for IBD
Q102 Gastrointestinal
What is the mechanism of Giardia-induced
diarrhea?

A) Enterotoxin activity
B) Local epithelial disruption
C) Mucosal erosion
Q102 Gastrointestinal
What is the mechanism of Giardia-induced
diarrhea?

A) Enterotoxin activity
B) Local epithelial disruption
C) Mucosal erosion
Q102 Gastrointestinal
A) Enterotoxin activity - C diff; watery, not oily stools; E coli enterotoxin
activity also causes a watery diarrhea ("travelers") lasting <5 days
B) Local epithelial disruption - disruption of epithelial tight junctions
between small intestine enterocytes, leading to malabsorption; nonbloody,
foul-smelling, oily diarrhea that can occur 1-2 weeks after exposure (e.g.,
traveling, contaminated food/water, rivers/lakes)
C) Mucosal erosion - UC involves mucosal inflammation and erosion of the
colon; prolonged diarrhea common; abdominal tenderness/anemia
Q103 Gastrointestinal
Acute pain and tenderness in the LUQ + nausea +
fever in a patient + (embolism or thrombosis) =
Q103 Gastrointestinal
Acute pain and tenderness in the LUQ + nausea +
fever in a patient + (embolism or thrombosis) =

Splenic infarction!
Q103 Gastrointestinal
Acute pain and tenderness in the LUQ + nausea +
fever in a patient + (embolism or thrombosis) =
Note the wedge-shaped splenic infarction.
Splenic infarction! Most cases of SI arise in the setting of
either:
1) thrombosis (due to hypercoaguble state
such as cancer or SLE)
2) embolism (e.g., afib, endocarditis)
3) hemoglobinopathy (e.g., sickle cell)
Q104 Gastrointestinal
A person undergoes surgery and several days later
develops RUQ pain, fever, and leukocytosis;
imaging shows a distended gallbladder with no
gallstones. What is the diagnosis?
Q104 Gastrointestinal
Acalculous cholecystitis
Acalculous cholecystitis results from gallbladder stasis and
ischemia, which then cause a local inflammatory response in
the gallbladder wall; seen in patients with trauma or recent
surgery or illness, or prolonged fasting
Tx: antibiotics and percutaneous cholecystostomy (i.e.,
drainage); cholecystectomy once the patient's medical
condition improves
Q105 Gastrointestinal
Painful, erythematous nodules on the shins is most
associated with:

A) Celiac
B) IBD
Q105 Gastrointestinal
Painful, erythematous nodules on the shins is most
associated with:

A) Celiac
B) IBD
Q105 Gastrointestinal
Painful, erythematous nodules on the shins is most
associated with:
A) Celiac - dermatitis herpetiformis (itchy papules on elbows,
forearms, and buttocks)
B) IBD - erythema nodosum (painful erythematous nodules on the
shins) is seen in IBD (e.g., patients have chronic diarrhea &
weight loss); more common in Crohn than in UC; EN mirrors IBD
disease activity - i.e., it worsens during IBD flares
Q106 Gastrointestinal
A pregnant woman presents with hematochezia,
abdominal pain, and tenesmus (i.e., fecal urgency
followed by straining and an inability to defecate).
The most likely diagnosis is:

A) Internal hemorrhoids
B) Ulcerative colitis
Q106 Gastrointestinal
A pregnant woman presents with hematochezia,
abdominal pain, and tenesmus (i.e., fecal urgency
followed by straining and an inability to defecate).
The most likely diagnosis is:

A) Internal hemorrhoids
B) Ulcerative colitis
Q106 Gastrointestinal
A) Internal hemorrhoids - very common in pregnancy and present
with streaky bloody stools in the setting of constipation; would not
explain tenesmus
B) Ulcerative colitis - pregnancy can worsen UC due to placental
cytokine colonic inflammation - leading to severe hematochezia and
anemia

A) Internal hemorrhoids
B) Ulcerative colitis
Q107 Gastrointestinal
A patient in the ICU for several days develops an
occult GI hemorrhage; he has no abdominal pain,
nausea, vomiting, hematemesis, diarrhea or
hematochezia. What is the diagnosis?

A) Acute colonic ischemia


B) Stress induced ulcer
Q107 Gastrointestinal
A patient in the ICU for several days develops an
occult GI hemorrhage; he has no abdominal pain,
nausea, vomiting, hematemesis, diarrhea or
hematochezia. What is the diagnosis?

A) Acute colonic ischemia


B) Stress induced ulcer
Q107 Gastrointestinal
Stress induced ulcer
-common in ICU patients; develop within hours to days of severe
physiologic stress
-multifactorial etiology (likely involves splanchnic hypoperfusion &
accumulation of uremic toxins that impair formation of the protective
mucosal layer around the stomach, allowing for mucosal injury and
bleeding)
-stress ulcers typically form in the proximal stomach and duodenum;
may be painless GI bleeding
-tx: with PPI's and close monitoring; endoscopy if significant bleed
Q108 Gastrointestinal
Porcelain gallbladder is a risk for:

A) gallbladder adenocarcinoma
B) anaphylactic shock
Q108 Gastrointestinal
Porcelain gallbladder is a risk for:

A) gallbladder adenocarcinoma
B) anaphylactic shock
Q108 Gastrointestinal
Porcelain gallbladder is a risk for:

A) gallbladder adenocarcinoma (porcelain gallbladder


describes the calcium laden gallbladder wall with brittle
consistency; pathogenesis unclear; assoicated with acute
cholecystitis; 4% risk of adenocarcinoma; consider
cholecystectomy, esp. if symptomatic)
B) anaphylactic shock - ruptured echinococcal cyst;
usually appears on CT as cystic liver lesion
Q109 Gastrointestinal
A woman presents scleral icterus and hyperbilirubinemia and
a positive urine bilirubin assay. Her symptoms are intermittent
and her LFT's are otherwise normal. Liver biopsy shows dark
granular pigment accumulation within hepatocytes. What is
the mechanism of her illness?

A) Decreased bilirubin glucuronidation


B) Impaired hepatocyte bilirubin excretion
Q109 Gastrointestinal
A woman presents scleral icterus and hyperbilirubinemia and
a positive urine bilirubin assay. Her symptoms are intermittent
and her LFT's are otherwise normal. Liver biopsy shows dark
granular pigment accumulation within hepatocytes. What is
the mechanism of her illness?

A) Decreased bilirubin glucuronidation


B) Impaired hepatocyte bilirubin excretion
Q109 Gastrointestinal
B) Impaired hepatocyte bilirubin excretion
-since she has ↑bilirubin in her urine, she has conjugated hyperbilirubinemia
(since unconjugated bilirubin can't be excreted in urine); her intermittent
symptoms and otherwise normal LFT's suggest benign hereditary
hyperbilirubinemia (Dubin Johnson syndrome), a condition caused by a
defect in a hepatocyte transporter protein responsible for bilirubin excretion
into the biliary system); liver biopsy shows a grossly black liver and dark,
granular pigment accumulation within hepatocytes (but this is not required
for diagnosis); DJS has no long-term sequelae; treatment is not necessary.
Q110 Gastrointestinal
If there is a concern that a patient may have a post-bariatric
surgery anastomotic leak (fever, severe epigastric abdominal
pain, unrelated to eating, worsening in intensity over time),
the next step would be:

A) Abdominal CT scan with contrast


B) EGD
C) HIDA scan
Q110 Gastrointestinal
If there is a concern that a patient may have a post-bariatric
surgery anastomotic leak (fever, severe epigastric abdominal
pain, unrelated to eating, worsening in intensity over time),
the next step would be:

A) Abdominal CT scan with contrast


B) EGD
C) HIDA scan
Q110 Gastrointestinal
A) Abdominal CT scan with contrast
-sometimes the gastrojejunal or jejunojejunal anastomosis
breaks down (usually within 1 week of surgery); symptoms
can include fever, abdominal pain, and tachycardia; get CT or
upper GI series if suspected; tx: with urgent surgery
(otherwise, ↑mortality)
B) EGD - contraindicated in anastomotic leak suspected!
C) HIDA scan - used to diagnose cholecystitis
Q110 Gastrointestinal
Several days after undergoing laparoscopic appendectomy, a
woman develops fever, abdominal pain, and vomiting.
Leukocyte count is 25,000/mm3. What is the diagnosis?

A) Subphrenic abscess
B) Mechanical bowel obstruction
Q110 Gastrointestinal
Several days after undergoing laparoscopic appendectomy, a
woman develops fever, abdominal pain, and vomiting.
Leukocyte count is 25,000/mm3. What is the diagnosis?

A) Subphrenic abscess
B) Mechanical bowel obstruction
Q110 Gastrointestinal
Several days after undergoing laparoscopic appendectomy, a
woman develops fever, abdominal pain, and vomiting.
Leukocyte count is 25,000/mm3. What is the diagnosis?
A) Subphrenic abscess - a complication of appendectomy
(especially with laparoscopic appendectomy); Dx: CT scan of
the abdomen; Tx: drainage and antibiotics
B) Mechanical bowel obstruction - e.g., adhesion from
previous surgery; adhesions take months to form
Q111 Gastrointestinal
HBsAg Negative
Anti-HBs Positive
Anti-HBc Positive
HBeAg Negative
Anti-HBe Negative

What do these labs indicate?


A) Recovery phase of HBV infection
B) Resolved HBV infection
C) Vaccination against HBV infection
Q111 Gastrointestinal
HBsAg Negative
Anti-HBs Positive
Anti-HBc Positive
HBeAg Negative
Anti-HBe Negative

What do these labs indicate?


A) Recovery phase of HBV infection
B) Resolved HBV infection
C) Vaccination against HBV infection
Q111 Gastrointestinal
HBsAg Negative
Anti-HBs Positive
Anti-HBc Positive
HBeAg Negative
Anti-HBe Negative

What do these labs indicate?


A) Recovery phase of HBV infection - anti-HBs, anti-HBc, and anti-
HBe
B) Resolved HBV infection
C) Vaccination against HBV infection - anti-HBs (but not anti-HBc)
Q112 Gastrointestinal
Screening colonoscopy should be offered to
patients with UC at 8 years after diagnosis and
then:

A) every 1-3 years


B) every 5 years
Q112 Gastrointestinal
Screening colonoscopy should be offered to
patients with UC at 8 years after diagnosis and
then:

A) every 1-3 years


B) every 5 years
Q112 Gastrointestinal
Screening colonoscopy should be offered to
patients with UC at 8 years after diagnosis and
then:
A) every 1-3 years - due to increased risk for
colorectal cancer
B) every 5 years - people w/o UC but first
degree relative who got CRC before age 60
Q113 Gastrointestinal
A patient presents with fever, jaundice, and RUQ;
ALP and bilirubin are markedly elevated. Which
finding is associated with the diagnosis?

A) "beads on a string"
B) pseudocysts
C) Reynolds pentad
Q113 Gastrointestinal
A patient presents with fever, jaundice, and RUQ;
ALP and bilirubin are markedly elevated. Which
finding is associated with the diagnosis?

A) "beads on a string"
B) pseudocysts
C) Reynolds pentad
Q113 Gastrointestinal
A patient presents with fever, jaundice, and RUQ;
ALP and bilirubin are markedly elevated. Which
finding is associated with the diagnosis?

A) "beads on a string" - PSC; acute illness unlikely


B) pseudocysts
Charcot triad of acute cholangitis =
C) Reynolds pentad; fever, jaundice, RUQ pain
Reynolds = +confusion +hypotension
Q114 Gastrointestinal
First-line treatment for mild UC limited to the
rectosigmoid is:

A) mesalamine enema
B) TNF-alpha inhibitor
Q114 Gastrointestinal
First-line treatment for mild UC limited to the
rectosigmoid is:

A) mesalamine enema
B) TNF-alpha inhibitor
Q114 Gastrointestinal

A) mesalamine enema - mild UC is defined as <4 watery


bowel movements a day with intermittent hematochezia and
no anemia; tx first with 5-ASA meds (sulfasalazine,
mesalamine) [mesalamine enema preferred if confined to
the rectosigmoid])
B) TNF-alpha inhibitor - 1st line for moderate to severe UC
(>6 bowel movements a day, frequent hematochezia,
anemia, elevated inflammatory markers)
Q115 Gastrointestinal
F\

Fascial dehiscence with potential evisceration is


managed with:

A) Abdominal binders
B) Emergency surgery
Q115 Gastrointestinal
F\

Fascial dehiscence with potential evisceration is


managed with:

A) Abdominal binders
B) Emergency surgery
Q115 Gastrointestinal
F\

Fascial dehiscence with potential evisceration is managed


with:
A) Abdominal binders - temporarily for fascial dehiscence;
contraindicated in patients with BOWEL evisceration due to
risk of perforation
B) Emergency surgery - fascial dehiscence is typically
seen in the first 1-2 weeks after surgery (but sometimes up
to 30 days); require surgery because of risk of herniation
and strangulation
Q116 Gastrointestinal
Treatment for actively bleeding esophageal varices in a
patient (Hb = 10 g/dL, Platelets = 73,000/mm3) involves
hemodynamic support, prophylactic antibiotics, and _.

A) Octreotide
B) Recombinant factor 7a
C) Platelets
D) Packed red blood cells
Q116 Gastrointestinal
Treatment for actively bleeding esophageal varices in a
patient (Hb = 10 g/dL, Platelets = 73,000/mm3) involves
hemodynamic support, prophylactic antibiotics, and _.

A) Octreotide
B) Recombinant factor 7a
C) Platelets
D) Packed red blood cells
Q116 Gastrointestinal
Treatment for actively bleeding esophageal varices in a
patient (Hb = 10 g/dL, Platelets = 73,000/mm3) involves
hemodynamic support, prophylactic antibiotics, and _.

A) Octreotide - a somatostatin analog; vasoconstriction


B) Recombinant factor 7a - for hemophilia
C) Platelets - only if platelet count less than 50,000
D) Packed red blood cells - if Hg below 7 g/dL
Q116 Gastrointestinal
Treatment for actively bleeding esophageal varices in a
patient (Hb = 10 g/dL, Platelets = 73,000/mm3) involves
hemodynamic support, prophylactic antibiotics, and _.

A) Octreotide - a somatostatin analog; vasoconstriction


AND HOW DO YOU PREVENT VARICES???
Q116 Gastrointestinal
Treatment for actively bleeding esophageal varices in a
patient (Hb = 10 g/dL, Platelets = 73,000/mm3) involves
hemodynamic support, prophylactic antibiotics, and _.

A) Octreotide - a somatostatin analog; vasoconstriction


AND HOW DO YOU PREVENT VARICES???
BETA BLOCKERS!!!
Q117 Gastrointestinal
Symptomatic anemia with macrocytic RBC's,
hypersegmented neutrophils, and normal methylmalonic
acid levels are seen in:

A) Cobalamin deficiency
B) Folate deficiency
Q117 Gastrointestinal
Symptomatic anemia with macrocytic RBC's,
hypersegmented neutrophils, and normal methylmalonic
acid levels are seen in:

A) Cobalamin deficiency
B) Folate deficiency
Q117 Gastrointestinal
Symptomatic anemia with macrocytic RBC's,
hypersegmented neutrophils, and normal methylmalonic
acid levels are seen in:

A) Cobalamin deficiency - MMA would be high


B) Folate deficiency - folate is found in leafy vegetables and
meats; deficiency rare in developed countreies; may be seen
in severe alcoholic
Q118 Gastrointestinal
An 8-year-old boy has a family history of colon polyps and a
germline mutation in the adenomatous polyposis coli tumor
suppressor gene. What is the next step?

A) Total proctocolectomy
B) Frequent colonoscopy
Q118 Gastrointestinal
An 8-year-old boy has a family history of colon polyps and a
germline mutation in the adenomatous polyposis coli tumor
suppressor gene. What is the next step?

A) Total proctocolectomy
B) Frequent colonoscopy
Q118 Gastrointestinal
An 8-year-old boy has a family history of colon polyps and a
germline mutation in the adenomatous polyposis coli tumor
suppressor gene. What is the next step?

A) Total proctocolectomy
B) Frequent colonoscopy
-patients almost universally develop CRC before age 40 if
untreated; wait until late teen years for proctocolectomy
(unless severe dysplasia)
Q119 Gastrointestinal
A woman with a history of cervical cancer treated with
radiation therapy now has hematochezia. Colonoscopy
reveals mucosal pallor, friability, and telangectasias confined
to the rectum. What is the diagnosis?

A) Crohn disease
B) Radiation proctitis
Q119 Gastrointestinal
A woman with a history of cervical cancer treated with
radiation therapy now has hematochezia. Colonoscopy
reveals mucosal pallor, friability, and telangectasias confined
to the rectum. What is the diagnosis?

A) Crohn disease
B) Radiation proctitis
Q119 Gastrointestinal
A woman with a history of cervical cancer treated with
radiation therapy now has hematochezia. Colonoscopy
reveals mucosal pallor, friability, and telangectasias confined
to the rectum. What is the diagnosis?

A) Crohn disease
B) Radiation proctitis - occurs due to damage to the rectal
epithelium associated with pelvic radiation therapy (acute vs.
CHRONIC - telangiectasias/bleeding)
Q120 Gastrointestinal
A 34-year-old woman presents with a 4-month history of
intermittent retrosternal pain; episodes last 10 minutes, and are
instigated by stress and cold/hot food. ECG is normal. Sublingual
nitroglycerin alleviates some pain, but a stress test does not
produce symptoms or abnormal EKG changes. Upper GI is
unremarkable. What is the next step?

A) 24-hour pH monitoring
B) Esophageal motility studies
Q120 Gastrointestinal
A 34-year-old woman presents with a 4-month history of
intermittent retrosternal pain; episodes last 10 minutes, and are
instigated by stress and cold/hot food. ECG is normal. Sublingual
nitroglycerin alleviates some pain, but a stress test does not
produce symptoms or abnormal EKG changes. Upper GI is
unremarkable. What is the next step?

A) 24-hour pH monitoring
B) Esophageal motility studies
Q120 Gastrointestinal
A 34-year-old woman presents with a 4-month history of intermittent retrosternal
pain; episodes last 10 minutes, and are instigated by stress and cold/hot food.
ECG is normal. Sublingual nitroglycerin alleviates some pain, but a stress test
does not produce symptoms or abnormal EKG changes. Upper GI is
unremarkable. What is the next step?

A) 24-hour pH monitoring - for GERD


B) Esophageal motility studies - to assess for diffuse esophageal
spasm; nitrates relax not only myocytes but also smooth muscle
cells of the esophagus!
Q121 Gastrointestinal
A man sustains BAT during a car accident. CT scan
reveals the following.

What is the next step?

A) Peritoneal lavage
B) Surgical exploration
Q121 Gastrointestinal
A man sustains BAT during a car accident. CT scan
reveals the following.

What is the next step?

A) Peritoneal lavage
B) Surgical exploration
Q121 Gastrointestinal
A man sustains BAT during a car accident. CT scan
reveals the following.

What is the next step?

A) Peritoneal lavage - no need, there is


clear evidence of GI perforation
B) Surgical exploration - free air!
Q122 Gastrointestinal
A 63-year-old woman complains of 3 months of
constipation and crampy left lower abdomen pain. The pain
is not affected by bowel movements. She doesn't have a
history of constipation. What is the best next step?

A) Polyethylene glycol
B) Pelvic ultrasound
Q122 Gastrointestinal
A 63-year-old woman complains of 3 months of
constipation and crampy left lower abdomen pain. The pain
is not affected by bowel movements. She doesn't have a
history of constipation. What is the best next step?

A) Polyethylene glycol
B) Pelvic ultrasound
Q122 Gastrointestinal
A 63-year-old woman complains of 3 months of
constipation and crampy left lower abdomen pain. The pain
is not affected by bowel movements. She doesn't have a
history of constipation. What is the best next step?

A) Polyethylene glycol
B) Pelvic ultrasound - concerning that the constipation onset is at
an older age (need to consider ovarian cancer)
Q123 Gastrointestinal
A pregnant woman in her third trimester has elevated total
bile acids, elevated aminotransferases and pruritis that is
worse on her hands and feet. At 37-weeks it is discovered
that the fetus has no cardiac activity. Platelets = 140,000.
What is the diagnosis?

A) Intrahepatic cholestasis of pregnancy


B) Acute fatty liver of pregnancy
Q123 Gastrointestinal
A pregnant woman in her third trimester has elevated total
bile acids, elevated aminotransferases and pruritis that is
worse on her hands and feet. At 37-weeks it is discovered
that the fetus has no cardiac activity. Platelets = 140,000.
What is the diagnosis?

A) Intrahepatic cholestasis of pregnancy


B) Acute fatty liver of pregnancy
Q123 Gastrointestinal
A) Intrahepatic cholestasis of pregnancy
ICP typically occurs in 3rd trimester because ↑estro/prog →
↓bile excretion. → ↑bile acids which accumulate in the liver
(aminotransferases) and skin (pruritis); bile acids can cross
the placenta and potentially lead to fetal demise. tx:
ursodeoxycholic acid, to decrease bile acid levels; delivery
at 37 weeks
B) Acute fatty liver of pregnancy - present with liver failure
signs - scleral icterus, platelet count < 100,000
Q124 Gastrointestinal
Can a lady on methadone maintenance therapy
breastfeed?

A) Yes
B) No
Q124 Gastrointestinal
Can a lady on methadone maintenance therapy
breastfeed?

A) Yes
B) No
Q124 Gastrointestinal
Breastfeeding Contraindications
-Active untreated tuberculosis
-HIV infection (in general)
-Herpetic breast lesions
-Active varicella infection
-Chemotherapy/radiation therapy
-Active substance use disorder
-Galactosemia (in infant)
Q125 Gastrointestinal
A 4.1 cm multilocular cyst is found incidentally in the
head of the pancreas of a man. The main pancreatic
duct is somewhat dilated. What is the next step?

A) Reassurance
B) Biopsy
Q125 Gastrointestinal
A 4.1 cm multilocular cyst is found incidentally in the
head of the pancreas of a man. The main pancreatic
duct is somewhat dilated. What is the next step?

A) Reassurance
B) Biopsy
Q125 Gastrointestinal
A 4.1 cm multilocular cyst is found incidentally in the
head of the pancreas of a man. The main pancreatic
duct is somewhat dilated. What is the next step?
A) Reassurance
B) Biopsy
very low risk of malignancy in pancreatic cysts. Concerning features include
large size (>3cm), thickened/irregular, calcifications, and main pancreatic
duct involvement.
Q126 Gastrointestinal
What is the initial managmement of uncomplicated
hemorrhoids?

A) High fiber diet and follow up


B) Colonoscopy
C) Topical nifedipine
Q126 Gastrointestinal
What is the initial managmement of uncomplicated
hemorrhoids?

A) High fiber diet and follow up


B) Colonoscopy
C) Topical nifedipine
Q126 Gastrointestinal
A) High fiber diet and follow up
-hemorrhoid diagnosis is based on clinical findings but anoscopy is
useful if they are not palpalbe on exam; initial management
includes increased fluids and fiber, and reduction in fat/alcohol
B) Colonoscopy - only if rectal bleeding or a concern for
malignancy (e.g., age 45 with a first degree relative with early onset
colon cancer)
C) Topical nifedipine - to reduce anal sphincter pressure in
patients with anal fissures (severe pain; sometimes bleeding)
Q127 Gastrointestinal
A patient with ulcerative colitis develops 2 days of
abdominal cramps and voluminous watery diarrhea
several times a day. What is the next step?

A) TNF-alpha inhibitor therapy


B) Stool testing for C diff
Q127 Gastrointestinal
A patient with ulcerative colitis develops 2 days of
abdominal cramps and voluminous watery diarrhea
several times a day. What is the next step?

A) TNF-alpha inhibitor therapy


B) Stool testing for C diff
Q127 Gastrointestinal
A patient with ulcerative colitis develops 2 days of
abdominal cramps and voluminous watery diarrhea
several times a day. What is the next step?
A) TNF-alpha inhibitor therapy for refractory UC (but
UC causes small volume bloody diarrhea)
B) Stool testing for C diff - not only antibiotics, but
also IBD can increase susceptibility to C diff
Q128 Gastrointestinal
A 25-year-old thin female as frequent watery bowel
movements. Labs show hypokalemia and metabolic
acidosis. A dark brown discoloration of the proximal
colon is seen on colonscopy. Which finding will be
positive in this patient?

A) Elevated VIP
B) Positive stool laxative screen
Q128 Gastrointestinal
A 25-year-old thin female as frequent watery bowel
movements. Labs show hypokalemia and metabolic
acidosis. A dark brown discoloration of the proximal
colon is seen on colonscopy. Which finding will be
positive in this patient?

A) Elevated VIP
B) Positive stool laxative screen
Q128 Gastrointestinal
A) Elevated VIP
B) Positive stool laxative screen - concern for
factitious diarrhea (diarrhea associated with laxative
abuse - leads to hypokalemia as K+ is lost in the stool
- can lead to a metabolic alkalosis); dx: bisacodyl or
polyethyene in the stool; colonoscopy shows
melanosis coli (dark brown discoloration of the colon
with pale patches of lymph follicles)
Q129 Gastrointestinal
What is the diagnosis of this 1-day-old baby?

A) Duodenal atresia
B) Jejunal atresia
C) Necrotizing enterocolitis
Q129 Gastrointestinal
What is the diagnosis of this 1-day-old baby?

A) Duodenal atresia
B) Jejunal atresia
C) Necrotizing enterocolitis
Q129 Gastrointestinal
What is the diagnosis of this 1-day-old baby?

A) Duodenal atresia - "double bubble"


B) Jejunal atresia - "triple bubble"
C) Necrotizing enterocolitis
Q130 Gastrointestinal
Which form of nutrition is optimal for severe burn
injury patients?

A) Enteral
B) Parenteral
Q130 Gastrointestinal
Which form of nutrition is optimal for severe burn
injury patients?

A) Enteral
B) Parenteral
Q130 Gastrointestinal
Which form of nutrition is optimal for severe burn
injury patients?

A) Enteral - preferred because it's associated with


multiple clinical benefits including: maintenance of gut
integrity, reduced rates of sepsis and infections,
decreased mortality
B) Parenteral - IV
Q131 Gastrointestinal
A man gets crushed by a motorcycle. He now shows signs of
hypovolemic shock (severe hypotension, tachycardia). CXR
shows an 8th and 9th rib fracture on the right and a 7th rib on
the left. What is the most likely diagnosis?

A) Duodenal hematoma
B) Liver laceration
C) Splenic hematoma
Q131 Gastrointestinal
A man gets crushed by a motorcycle. He now shows signs of
hypovolemic shock (severe hypotension, tachycardia). CXR
shows an 8th and 9th rib fracture on the right and a 7th rib on
the left. What is the most likely diagnosis?

A) Duodenal hematoma
B) Liver laceration
C) Splenic hematoma
Q131 Gastrointestinal
A man gets crushed by a motorcycle. He now shows signs of
hypovolemic shock (severe hypotension, tachycardia). CXR
shows an 8th and 9th rib fracture on the right and a 7th rib on
the left. What is the most likely diagnosis?

A) Duodenal hematoma - rapid decompensation uncommon


B) Liver laceration - frequently injured in BAT
C) Splenic hematoma - rapid decompensation uncommon
Q132 Gastrointestinal
In alcoholic liver hepatitis, which lab values are
expected?

A) AST = 235, ALT = 110; GGT elevated


B) AST = 2500, ALT = 1050; GGT elevated
Q132 Gastrointestinal
In alcoholic liver hepatitis, which lab values are
expected?

A) AST = 235, ALT = 110; GGT elevated


B) AST = 2500, ALT = 1050; GGT elevated
Q132 Gastrointestinal
In alcoholic liver hepatitis, which lab values are
expected?

always less than 500!


A) AST = 235, ALT = 110; GGT elevated
B) AST = 2500, ALT = 1050; GGT elevated
Q133 Gastrointestinal
What is the mechanism of physiologic jaundice?

A) Low UGT activity levels


B) Rapid conversion of bilirubin to urobilinogen
Q133 Gastrointestinal
What is the mechanism of physiologic jaundice?

A) Low UGT activity levels


B) Rapid conversion of bilirubin to urobilinogen
Q133 Gastrointestinal
What is the mechanism of physiologic jaundice?
A) Low UGT activity levels
-at birth fetal RBCs are increased with a shortened life span,
resulting in high RBC turnover and ↑bilirubin production.
Hepatic bilirubin clearance is decreased because UGT activity
doesn't reach adult levels until age 2 weeks; and, low bacterial
load in gut results in SLOWER conversion of bilirubin to
urobilinogen for fecal excretion
B) Rapid conversion of bilirubin to urobilinogen
Q134 Gastrointestinal
A 40-year-old woman has had 2 days of abdominal pain,
nausea, and vomiting. She hasn't pooped or flatulated in 3
days. Temp = 102.1F. BP = 90/63. The abdomen is
distended. Bowel sounds are decreased. What is the next
step?

A) Urgent surgery
B) Small-bowel follow through
C) Broad spectrum antibiotics
Q134 Gastrointestinal
A 40-year-old woman has had 2 days of abdominal pain,
nausea, and vomiting. She hasn't pooped or flatulated in 3
days. Temp = 102.1F. BP = 90/63. The abdomen is
distended. Bowel sounds are decreased. What is the next
step?

A) Urgent surgery
B) Small-bowel follow through
C) Broad spectrum antibiotics
Q134 Gastrointestinal
A 40-year-old woman has had 2 days of abdominal pain,
nausea, and vomiting. She hasn't pooped or flatulated in 3
days. Temp = 102.1F. BP = 90/63. The abdomen is
distended. Bowel sounds are decreased. What is the next
step?

A) Urgent surgery - SBO with hemodynamic instability


B) Small-bowel follow through - not appropriate here
C) Broad spectrum antibiotics - she's deteriorating!
Q135 Gastrointestinal
A woman on long term NSAIDs has suspected peptic
ulcers; she now has fever and signs of peritonitis. What
is the next step?

A) NSAID avoidance and oral PPI's


B) Upright x-ray of chest and abdomen
Q135 Gastrointestinal
A woman on long term NSAIDs has suspected peptic
ulcers; she now has fever and signs of peritonitis. What
is the next step?

A) NSAID avoidance and oral PPI's


B) Upright x-ray of chest and abdomen
Q135 Gastrointestinal
A woman on long term NSAIDs has suspected peptic ulcers; she now has
fever and signs of peritonitis. What is the next step?

A) NSAID avoidance and oral PPI's - this is part of uncomplicate PUD


management - not in this patient who is unstable (and should remain NPO
while awaiting definitive management)
B) Upright x-ray of chest and abdomen - peptic ulcer disease can be
complicated by perforation which causes acute-onset, severe pain and a
systemic response (fever, tachycardia, peritonitis); upright X-ray of the chest
and abdomen can confirm the diagnosis of perforation by showing free air
under the diaphgram
Q136 Gastrointestinal
A woman has recurrent abdominal pain and dyspepsia
some time after her cholecystectomy. Her LFT's are
elevated and abdominal ultrasound shows a mild
dilation of the common bile duct. What is the next step?

A) ERCP
B) Ursodeoxycholic acid therapy
Q136 Gastrointestinal
A woman has recurrent abdominal pain and dyspepsia
some time after her cholecystectomy. Her LFT's are
elevated and abdominal ultrasound shows a mild
dilation of the common bile duct. What is the next step?

A) ERCP
B) Ursodeoxycholic acid therapy
Q136 Gastrointestinal
A) ERCP - for postcholecystectomy syndrome
(recurrent/constant abdominal pain and/or dyspepsia
that occurs months or years post-op; ERCP to evaluate
the biliary tree can confirm the diagnosis [e.g., biliary
stricture] and guide therapy)
B) Ursodeoxycholic acid therapy - slows progression of
PBC, and can treat cholesterol gallstones (not for PCS)
Q137 Gastrointestinal
A patient with a malignant gastric outlet obstruction gets
a feeding tube. Now he has signs of hypophosphatemia,
hypokalemia, and intermittent arrhythmias. What is the
cause of this condition?

A) Hypermetabolic syndrome
B) Replenishment of nutrients
Q137 Gastrointestinal
A patient with a malignant gastric outlet obstruction gets
a feeding tube. Now he has signs of hypophosphatemia,
hypokalemia, and intermittent arrhythmias. What is the
cause of this condition?

A) Hypermetabolic syndrome
B) Replenishment of nutrients
Q137 Gastrointestinal
A patient with a malignant gastric outlet obstruction gets
a feeding tube. Now he has signs of hypophosphatemia,
hypokalemia, and intermittent arrhythmias. What is the
cause of this condition?
A) Hypermetabolic syndrome - severe burns, sepsis
B) Replenishment of nutrients - refeeding syndrome;
occurs in chronically malnourished patients (e.g.,
anorexia nervosa, malignancy)
Q138 Gastrointestinal
A 2-year-old boy has intermittent abdominal pain and
vomiting that began several hours ago. Ultrasound of
the abdomen is shown. What is the next step?

A) Urgent surgery
B) Air enema
Q138 Gastrointestinal
A 2-year-old boy has intermittent abdominal pain and
vomiting that began several hours ago. Ultrasound of
the abdomen is shown. What is the next step?

A) Urgent surgery
B) Air enema
Q138 Gastrointestinal
A 2-year-old boy has intermittent abdominal pain and
vomiting that began several hours ago. Ultrasound of
the abdomen is shown. What is the next step?

target sign is diagnostic


A) Urgent surgery
B) Air enema - for intussusception (presents in children
age 6-36 months)
Q139 Gastrointestinal
An 11-month-old girl has had bright red stools since
yesterday; PE shows diffuse abdominal tenderness.
Fecal testing is positive for blood. What is the
diagnosis?

A) Acute dysentery
B) Intussusception
Q139 Gastrointestinal
An 11-month-old girl has had bright red stools since
yesterday; PE shows diffuse abdominal tenderness.
Fecal testing is positive for blood. What is the
diagnosis?

A) Acute dysentery
B) Intussusception
Q139 Gastrointestinal
An 11-month-old girl has had bright red stools since yesterday;
PE shows diffuse abdominal tenderness. Fecal testing is
positive for blood. What is the diagnosis?
A) Acute dysentery
B) Intussusception - Intussusception is most common in
children age 6-36 months and causes episodic abdominal
pain, currant jelly stools, and lethargy. A palpable, sausage-
shaped abdominal mass is not always present.
Q140 Gastrointestinal
What is given to a patient with an acute GI bleed with
hemoglobin <7 g/dL (besides fluids)?

A) Nothing
B) FFP
C) Packed RBC's
Q140 Gastrointestinal
What is given to a patient with an acute GI bleed with
hemoglobin <7 g/dL (besides fluids)?

A) Nothing
B) FFP
C) Packed RBC's
Q140 Gastrointestinal
What is given to a patient with an acute GI bleed with
hemoglobin <7 g/dL (besides fluids)?

A) Nothing
B) FFP - contains clotting factors and plasma proteins;
indicated for severe coagulopathy (e.g., DIC)
C) Packed RBC's - increase oxygen carrying capacity; note: a
higher threshold <9 g/dL can be considered for ACS patients
Q141 Gastrointestinal
A 15-year-old boy has acute onset fever, abdominal pain, and
leukocytosis. He also has vomiting and watery diarrhea. The
abdominal pain used to be periumbilical and now is in the RLQ.
There is no rebound or rigidity. The boy's younger brother also
has diarrhea and abdominal pain. What is the diagnosis?

A) Campylobacter infection
B) Appendicitis
Q141 Gastrointestinal
A 15-year-old boy has acute onset fever, abdominal pain, and
leukocytosis. He also has vomiting and watery diarrhea. The
abdominal pain used to be periumbilical and now is in the RLQ.
There is no rebound or rigidity. The boy's younger brother also
has diarrhea and abdominal pain. What is the diagnosis?

A) Campylobacter infection
B) Appendicitis
Q141 Gastrointestinal
A 15-year-old boy has acute onset fever, abdominal pain, and
leukocytosis. He also has vomiting and watery diarrhea. The
abdominal pain used to be periumbilical and now is in the RLQ.
There is no rebound or rigidity. The boy's younger brother also
has diarrhea and abdominal pain. What is the diagnosis?

A) Campylobacter infection - pseudoappendicitis


B) Appendicitis - not associated with profuse watery then mucoid
diarrhea; wouldn't explain close contact
Q142 Gastrointestinal
Bowel obstruction in a patient with Crohn disease is most likely
due to:

A) Infliximab
B) Fibrotic stricture
C) Adynamic illeus
Q142 Gastrointestinal
Bowel obstruction in a patient with Crohn disease is most likely
due to:

A) Infliximab
B) Fibrotic stricture
C) Adynamic illeus
Q142 Gastrointestinal
Bowel obstruction in a patient with Crohn disease is most likely
due to:

A) Infliximab - reduces inflammation in Crohn's


B) Fibrotic stricture - strictures are a common complication of
Crohn disease (from severe inflammation); typically presents with
bilious vomiting and severe abdominal pain; high-pitched bowel
sounds; surgery may be required
C) Adynamic illeus - when small bowel motility disrupted (e.g.,
after surgery)
Q143 Gastrointestinal
What is the first step in a patient with ALP = 820 (and signs of
hyperbilirubinemia), but normal AST and ALT levels?

A) ERCP
B) Ultrasound
Q143 Gastrointestinal
What is the first step in a patient with ALP = 820 (and signs of
hyperbilirubinemia), but normal AST and ALT levels?

A) ERCP
B) Ultrasound
Q143 Gastrointestinal
What is the first step in a patient with ALP = 820 (and signs of
hyperbilirubinemia), but normal AST and ALT levels?

A) ERCP
B) Ultrasound - to assess hepatic
parenchyma and biliary ducts
Q144 Gastrointestinal
A post-op woman has three days of nausea and vomiting. She
hasn't pooped/flatulated since the surgery. She is
hemodynamically stable. X-ray is shown. In addition to fluids,
what is the next step?

A) Surgery
B) Bowel reset and serial exams
Q144 Gastrointestinal
A post-op woman has three days of nausea and vomiting. She
hasn't pooped/flatulated since the surgery. She is
hemodynamically stable. X-ray is shown. In addition to fluids,
what is the next step?

A) Surgery
B) Bowel reset and serial exams
Q144 Gastrointestinal
A post-op woman has three days of nausea and vomiting. She
hasn't pooped/flatulated since the surgery. She is
hemodynamically stable. X-ray is shown. In addition to fluids,
what is the next step?

A) Surgery
B) Bowel reset and serial exams - post-operative ileus, can last
more than 72 hours; typically self-resolving - management is
conservative
Q145 Gastrointestinal
A male who has anoreceptive intercourse has an indurated
erythematous mass near the anal orifice, associated with severe
constant anal pain and fever. What is the pathogenesis?

A) an infected occluded anal crypt gland


B) overstretching of the anal mucosa
Q145 Gastrointestinal
A male who has anoreceptive intercourse has an indurated
erythematous mass near the anal orifice, associated with severe
constant anal pain and fever. What is the pathogenesis?

A) an infected occluded anal crypt gland


B) overstretching of the anal mucosa
Q146 Gastrointestinal
A 62-year-old with a history of weight loss and fatigue
presents and CT scan of his liver is shown. What is the next
step?

A) alpha-fetoprotein
B) colonoscopy
Q146 Gastrointestinal
A 62-year-old with a history of weight loss and fatigue
presents and CT scan of his liver is shown. What is the next
step?

A) alpha-fetoprotein
B) colonoscopy
Q146 Gastrointestinal
A 62-year-old with a history of weight loss and fatigue
presents and CT scan of his liver is shown. What is the next
step?

A) alpha-fetoprotein
B) colonoscopy - weight loss, fatigue, and multiple liver
lesions on CT scan suggest metastatic disease (vs. primary
liver cancer); colorectal cancer is the primary source
Q147 Gastrointestinal
A 4-year-old boy has watery diarrhea for 2 days that
becomes bloody on the third day. Temp = 98.6F. Bowel
sounds are hyperactive. The abdomen is diffusely tender
without rebound. What is the diagnosis?

A) Infection with E coli


B) Infection with Entamoeba
C) Intussusception
Q147 Gastrointestinal
A 4-year-old boy has watery diarrhea for 2 days that
becomes bloody on the third day. Temp = 98.6F. Bowel
sounds are hyperactive. The abdomen is diffusely tender
without rebound. What is the diagnosis?

A) Infection with E coli


B) Infection with Entamoeba
C) Intussusception
Q147 Gastrointestinal
A) Infection with E coli - Shiga toxin (STEC) - watery
diarrhea, then bloody; patients are afebrile or have a low
grade fever; self resolves within a week but infection may be
complicated by HUS after 1-2 weeks
B) Infection with Entamoeba - 2 weeks of bloody diarrhea,
seen in travelers to resource limited countries
C) Intussusception - 6-36 months; episodic pain and currant
jelly stools; focal abdominal tenderness; watery stools would
not be expected
Q148 Gastrointestinal
A 57-year-old male has hepatomegaly, elevated liver
transaminases, and a very high serum ferritin. This patient
would benefit from:

A) Prednisone
B) Phlebotomy
Q148 Gastrointestinal
A 57-year-old male has hepatomegaly, elevated liver
transaminases, and a very high serum ferritin. This patient
would benefit from:

A) Prednisone
B) Phlebotomy
Q148 Gastrointestinal
A 57-year-old male has hepatomegaly, elevated liver transaminases, and
a very high serum ferritin. This patient would benefit from:

A) Prednisone
B) Phlebotomy - these findings are indicative of hemochromatosis
(autosomal recessive disorder caused by an HFE genetic mutation;
increased intestinal iron absorption; elevated trasnferrin indicated
excessive iron stores; dx: HFE genetic mutation testing; tx: phlebotomy)
Q149 Gastrointestinal
A 25-hour infant has not passed meconium; he has
bilious emesis and dilated loops of small bowel on
imaging. The baby most likely has:

A) intussusception
B) cystic fibrosis
Q149 Gastrointestinal
A 25-hour infant has not passed meconium; he has
bilious emesis and dilated loops of small bowel on
imaging. The baby most likely has:

A) intussusception
B) cystic fibrosis
Q149 Gastrointestinal
A 25-hour infant has not passed meconium; he has bilious
emesis and dilated loops of small bowel on imaging. The
baby most likely has:

A) intussusception - ages 6-36 months; abdominal pain,


vomiting, bloody stools
B) cystic fibrosis - this baby most likely has meconium
ileus (blockage of terminal ileum by meconium); about 90%
of such patients have cystic fibrosis
Q150 Gastrointestinal
A basketball player undergoes trauma to his abdomen which
is now distended and diffusely tender. FAST shows
intraperitoneal free fluid and the patient is tachycardic and
hypotensive. What is the most likely diagnosis?

A) Duodenal hematoma
B) Pancreatic transection
C) Splenic laceration
Q150 Gastrointestinal
A basketball player undergoes trauma to his abdomen which
is now distended and diffusely tender. FAST shows
intraperitoneal free fluid and the patient is tachycardic and
hypotensive. What is the most likely diagnosis?

A) Duodenal hematoma
B) Pancreatic transection
C) Splenic laceration
Q150 Gastrointestinal
A) Duodenal hematoma - no intraperitoneal free fluid
because bleeding remains confined in duodenal wall
B) Pancreatic transection - doesn't cause an acute effusion
of free fluid
C) Splenic laceration - commonly injured in BAT; can cause
contusion or hematoma; can be life threatening due to
spleen's high vascularity; initially management may be
nonoperative; if hemodynamic instability continues, may
require laparotomy and splenectomy
Q151 Gastrointestinal
A woman who works at a daycare has loose, foul-smelling
stools 5 times each day/night over the last two months. What
is the next step?

A) Reassurance
B) Stool antigen testing
Q151 Gastrointestinal
A woman who works at a daycare has loose, foul-smelling
stools 5 times each day/night over the last two months. What
is the next step?

A) Reassurance
B) Stool antigen testing
Q151 Gastrointestinal
A woman who works at a daycare has loose, foul-smelling
stools 5 times each day/night over the last two months. What
is the next step?
A) Reassurance
B) Stool antigen testing - unlikely IBS, because concerning
features include nocturnal symptoms (others include: age
onset > age 50, systemic symptoms, weight loss, elevated
inflammatory markers); Giardia has fecal oral transmission;
dx: SAT or PCR
Q152 Gastrointestinal
Average-risk adults should undergo CRC
screening beginning at age _.

A) 40
B) 45
B) 50
Q152 Gastrointestinal
Average-risk adults should undergo CRC
screening beginning at age _.

A) 40
B) 45
B) 50
Q152 Gastrointestinal
Average-risk adults should undergo CRC
screening beginning at age _.

A) 40 - high risk adults (e.g., FDR, high risk polyp)


B) 45 - average risk (& then every 10 years)
B) 50
Q153 Gastrointestinal
A patient has fever, leukocytosis, RUQ pain; here
is a CT scan of his liver. He hasn't traveled
recently. What should be done to diagnose him?

A) E hystolitica serology
B) Percutaneous aspiration
Q153 Gastrointestinal
A patient has fever, leukocytosis, RUQ pain; here
is a CT scan of his liver. He hasn't traveled
recently. What should be done to diagnose him?

A) E hystolitica serology
B) Percutaneous aspiration
Q153 Gastrointestinal

A) E hystolitica serology - hepatic abscess seen; but generally


in travelers to endemic areas, such as India or Mexico;
testing: serology or stool analysis
B) Percutaneous aspiration - for suspected pyogenic liver
abscess; CT shows hypoattenuating rounded lesion;
percutaneous aspiration is diagnostic & therapeutic
Q154 Gastrointestinal
After undergoing Nissen fundoplication, a patient
develops bloating, early satiety, postprandial emesis,
and weight loss. What is the next step?

A) Reassurance
B) Scintigraphic
gastric emptying scan
Q154 Gastrointestinal
After undergoing Nissen fundoplication, a patient
develops bloating, early satiety, postprandial emesis,
and weight loss. What is the next step?

A) Reassurance
B) Scintigraphic
gastric emptying scan
Q154 Gastrointestinal
After undergoing Nissen fundoplication, a patient
develops bloating, early satiety, postprandial emesis,
and weight loss. What is the next step?
A complication of NF (to treat GERD) is damage to
the vagal nerve (CN10), resulting in gastroparesis;
diagnosed by scintigraphic gastric emptying scan;
management of the gastroparesis is dietary
modification (e.g., small meals, low fat) and
promotility agents (e.g., metoclopramide)
Q154 Gastrointestinal
Biliary atresia is best managed with:

A) phototherapy
B) Kasai procedure
Q154 Gastrointestinal
Biliary atresia is best managed with:

A) phototherapy
B) Kasai procedure
Q154 Gastrointestinal
Biliary atresia is best managed with:

A) phototherapy
B) Kasai procedure
Q154 Gastrointestinal
Biliary Atresia
-biliary atresia involves fibrotic obliteration of the extrahepatic
bild ducts
-presents at age 2-8 weeks with hyperbilirubinemia - jaundice,
dark urine
-liver biopsy in all suspected patients; shows intrahepatic bile
proliferation and portal tract inflammation
Q155 Gastrointestinal
After surgical resesction of colon adenocarcinoma,
colonoscopy screening involves:

A) 1 year post surgery and thereafter every 3-5 years


B) no screening necessary
Q155 Gastrointestinal
After surgical resesction of colon adenocarcinoma,
colonoscopy screening involves:

A) 1 year post surgery and thereafter every 3-5 years


B) no screening necessary
Q155 Gastrointestinal
After surgical resesction of colon adenocarcinoma,
colonoscopy screening involves:

A) 1 year post surgery and thereafter every 3-5 years -


although most patients with stage 1 colon adenocarcinoma
are cured after resection, some develop recurrent
neoplastic colonic lesions
B) no screening necessary
Q156 Gastrointestinal
A patient with chronic constipation has pain and rectal
bleeding on defecation; rectal exam shows a posterior
mucosal tear of the anus. Besides stool softeners and sitz
baths, what is the best next step?

A) Biopsy
B) Topical lidocaine and nifedipine
Q156 Gastrointestinal
A patient with chronic constipation has pain and rectal
bleeding on defecation; rectal exam shows a posterior
mucosal tear of the anus. Besides stool softeners and sitz
baths, what is the best next step?

A) Biopsy
B) Topical lidocaine and nifedipine
Q156 Gastrointestinal
A patient with chronic constipation has pain and rectal
bleeding on defecation; rectal exam shows a posterior
mucosal tear of the anus. Besides stool softeners and sitz
baths, what is the best next step?
A) Biopsy - no need for biopsy of anal fissures
B) Topical lidocaine and nifedipine - lidocaine as a topical
anesthetic for pain; nifedipine as a vasodilator to increase
blood flow to the anal sphincter to facilitate healing
Q156 Gastrointestinal
Anal Fissure
-longitudinal tears in the anal canal distal to the dentate line
-most common at the posterior midline
-spasm of the sphincter contributes to the pain as tension is
created across the wound
-most are related to chronic constipation (but also frequent diarrhea
or anal intercourse)
-may be associated with a skin tag
-tx: dietary fiber, sitz baths, topical anesthetics, vasodilators
Q157 Gastrointestinal
An elderly patient has episodic maroon-colored stools
without associated abdominal or rectal pain, or nausea or
vomiting. Nothing was seen on colonscopy several months
earlier. What is the most likely diagnosis?

A) Colon cancer
B) Diverticulosis
C) Ischemic colitis
D) Angiodysplasia
Q157 Gastrointestinal
An elderly patient has episodic maroon-colored stools
without associated abdominal or rectal pain, or nausea or
vomiting. Nothing was seen on colonscopy several months
earlier. What is the most likely diagnosis?

A) Colon cancer
B) Diverticulosis
C) Ischemic colitis
D) Angiodysplasia
Q157 Gastrointestinal
An elderly patient has episodic maroon-colored stools
without associated abdominal or rectal pain, or nausea or
vomiting. Nothing was seen on colonscopy several months
earlier. What is the most likely diagnosis?

A) Colon cancer
B) Diverticulosis
C) Ischemic colitis
D) Angiodysplasia - dilated submucosal veins and AVM's
Q158 Gastrointestinal
A 49-year-old woman has chronic diarrhea, flushing, a
pounding sensation in her neck, weight loss, and valvular
heart disease with tricuspid regurgitation. What is the most
likely diagnosis?

A) Subacute endocarditis
B) Carcinoid syndrome
Q158 Gastrointestinal
A 49-year-old woman has chronic diarrhea, flushing, a
pounding sensation in her neck, weight loss, and valvular
heart disease with tricuspid regurgitation. What is the most
likely diagnosis?

A) Subacute endocarditis
B) Carcinoid syndrome
Q158 Gastrointestinal

A) Subacute endocarditis
B) Carcinoid syndrome - Carcinoid tumors are well-
differentiated neuroendocrine tumors; carcinoid tumors with
liver metastasis - as the tumor's hormones (serotonin,
histamine, VIP) are released into systemic circulation -
causing episodic flushing (85% of patients), and the other
aforementioned symptoms
Q159 Gastrointestinal
Steatorrhea in patients with chronic alcoholic
pancreatitis is managed with alcohol cessation and:

A) Antibiotics
B) Pancreatic enzyme supplementation
Q159 Gastrointestinal
Steatorrhea in patients with chronic alcoholic
pancreatitis is managed with alcohol cessation and:

A) Antibiotics
B) Pancreatic enzyme supplementation
Q160 Gastrointestinal
What is first-line treatment for bile-acid
diarrhea?

A) Cholestyramine
B) Budesonide
Q160 Gastrointestinal
What is first-line treatment for bile-acid
diarrhea?

A) Cholestyramine
B) Budesonide
Q160 Gastrointestinal
What is first-line treatment for bile-acid
diarrhea?

A) Cholestyramine
B) Budesonide
Q160 Gastrointestinal
A) Cholestyramine - BAD seen in ~10% of
patients after cholecystectomy, as bile acid
enters the SI more rapidly; cholestyramine (and
colestipol, colesevelam) bind bile acids and
prevent colonic irritation
B) Budesonide - for microscopic colitis,
triggered by medications (e.g., PPI's, NSAID)
Q161 Gastrointestinal
An 18-month-old can't tolerate solids. He is not
gaining weight. He suffers from eczima. What
is the most likely diagnosis?

A) EoE
B) Celiac disease
Q161 Gastrointestinal
An 18-month-old can't tolerate solids. He is not
gaining weight. He suffers from eczima. What
is the most likely diagnosis?

A) EoE
B) Celiac disease
Q161 Gastrointestinal
An 18-month-old can't tolerate solids. He is not
gaining weight. He suffers from eczima. What
is the most likely diagnosis?

A) EoE - consider in toddlers with poor feeding;


dx: endoscopy & biopsy
B) Celiac disease
Q162 Gastrointestinal
A 45-year-old woman presents with gradual
onset fatigue, pruritis, ↑ ALP, hepatomegaly
and xanthelasma. What is the next step?

A) MRI of the abdomen


B) Check anti-mitochondrial antibodies
Q162 Gastrointestinal
A 45-year-old woman presents with gradual
onset fatigue, pruritis, ↑ ALP, hepatomegaly
and xanthelasma. What is the next step?

A) MRI of the abdomen


B) Check anti-mitochondrial antibodies
Q163 Gastrointestinal
A 52-year-woman presents with a year of
postprandial abdominal pain, hypothyroidism, and
MCV = 105. What is the diagnosis?

A) Autoimmune metaplastic atrophic gastritis


B) Functional dyspepsia
Q163 Gastrointestinal
A 52-year-woman presents with a year of
postprandial abdominal pain, hypothyroidism, and
MCV = 105. What is the diagnosis?

A) Autoimmune metaplastic atrophic gastritis


B) Functional dyspepsia
Q163 Gastrointestinal
A 52-year-woman presents with a year of
postprandial abdominal pain, hypothyroidism, and
MCV = 105. What is the diagnosis?

A) Autoimmune metaplastic atrophic gastritis


characterized by the presence of antibodies toward parietal cells → atrophy
and metaplasia of the gastric corpus, hypochlorhydria, and unchecked gastrin
production; vitamin b12 deficiency → big MCV

B) Functional dyspepsia - no lab abnormalities


Q164 Gastrointestinal
A 64-year-old man has postprandial
abdominal pain and fecal occult blood is
positive. Gastroduodenoscopy shows a
gastric ulcer. Biopsies are consistent with
adenocarcinoma. What is the next step?

A) Get serologic markers


B) CT scan of the abdomen
Q164 Gastrointestinal
A 64-year-old man has postprandial
abdominal pain and fecal occult blood is
positive. Gastroduodenoscopy shows a
gastric ulcer. Biopsies are consistent with
adenocarcinoma. What is the next step?

A) Get serologic markers


B) CT scan of the abdomen
Q164 Gastrointestinal
A 64-year-old man has postprandial abdominal pain
and fecal occult blood is positive. Gastroduodenoscopy
shows a gastric ulcer. Biopsies are consistent with
adenocarcinoma. What is the next step?

A) Get serologic markers


B) CT scan of the abdomen - very
important to get tumor stage at time of
diagnosis, for prognosis and tx options
Q164 Gastrointestinal
Which image represents Hirschsprung disease?

A) B)
Q164 Gastrointestinal
Which image represents Hirschsprung disease?

A) B)
Q164 Gastrointestinal
Which image represents Hirschsprung disease?

A) B)
dilated bowel loops, absent rectal
air
Q165 Gastrointestinal
A football player undergoes BAT; abdominal imaging
shows free air in the retroperitoneum. Of the choices,
which is the most likely diagnosis?

A) Stomach laceration
B) Pancreatitis due to trauma
C) Duodenal tear
Q165 Gastrointestinal
A football player undergoes BAT; abdominal imaging
shows free air in the retroperitoneum. Of the choices,
which is the most likely diagnosis?

A) Stomach laceration
B) Pancreatitis due to trauma
C) Duodenal tear
Q165 Gastrointestinal
A football player undergoes BAT; abdominal imaging
shows free air in the retroperitoneum. Of the choices,
which is the most likely diagnosis?

A) Stomach laceration - intraperitoneal free air


B) Pancreatitis due to trauma - fluid
C) Duodenal tear - air
Q166 Gastrointestinal
A 37-year-old man has a foul-smelling anal
discharge and pruritis. There is no blood in the
stool but there is an indurated pustule-like lesion
close to the anal verge. The patient had an
episode of severe anal pain a few months ago
but it resolved in a few days. What is the next
step?

A) Stool test for ova


B) Surgery
Q166 Gastrointestinal
A 37-year-old man has a foul-smelling anal
discharge and pruritis. There is no blood in the
stool but there is an indurated pustule-like lesion
close to the anal verge. The patient had an
episode of severe anal pain a few months ago
but it resolved in a few days. What is the next
step?

A) Stool test for ova


B) Surgery
Q166 Gastrointestinal
A 37-year-old man has a foul-smelling anal
discharge and pruritis. There is no blood in the
stool but there is an indurated pustule-like lesion
close to the anal verge. The patient had an
episode of severe anal pain a few months ago
but it resolved in a few days. What is the next
step?

A) Stool test for ova


B) Surgery - for anorectal fistula
Q166 Gastrointestinal
Q167 Gastrointestinal
A 29-year-old man has intense midline chest pain for
several hours after vomiting for several hours. CXR shows
a widened mediastinum and a moderate pleural effusion.
The pleural fluid is yellow exudate with high amylase. What
is the diagnosis?

A) Acute pancreatitis
B) Esophageal perforation
Q167 Gastrointestinal
A 29-year-old man has intense midline chest pain for
several hours after vomiting for several hours. CXR shows
a widened mediastinum and a moderate pleural effusion.
The pleural fluid is yellow exudate with high amylase. What
is the diagnosis?

A) Acute pancreatitis
B) Esophageal perforation
Q167 Gastrointestinal
A) Acute pancreatitis
B) Esophageal perforation - Boerhaave; The perforation
allows GI contents to leak from the esophagus into the
mediastinum (causing widened mediastinum on CXR)
and/or into the pleural space (causing pleural effusion).
The inflammatory GI contents often cause severe chest
pain and a systemic inflammatory response (eg, fever,
tachycardia) that can quickly progress to septic shock and
death.
Q168 Gastrointestinal
A 17-year-old boy develops scleral icterus after surgery;
indirect bilirubin levels are elevated but LFT's are otherwise
normal. This is consistent with:

A) UDP glucuronosyltransferase deficiency


B) G6PD deficiency
Q168 Gastrointestinal
A 17-year-old boy develops scleral icterus after surgery;
indirect bilirubin levels are elevated but LFT's are otherwise
normal. This is consistent with:

A) UDP glucuronosyltransferase deficiency


B) G6PD deficiency
Q168 Gastrointestinal
A 17-year-old boy develops scleral icterus after surgery;
indirect bilirubin levels are elevated but LFT's are otherwise
normal. This is consistent with:

A) UDP glucuronosyltransferase deficiency - Gilbert


syndrome; benign disorder; recurrent episodes of mild
jaundice; precipitated by stressors (e.g., surgery)
B) G6PD deficiency
Q169 Gastrointestinal
An 11-year-old boy from Asia develops severe abdominal
pain and signs of SBO. He has no previous surgeries.
Eosinophils = 15%. What is the diagnosis?

A) Ascariasis
B) Adhesions
Q169 Gastrointestinal
An 11-year-old boy from Asia develops severe abdominal
pain and signs of SBO. He has no previous surgeries.
Eosinophils = 15%. What is the diagnosis?

A) Ascariasis
B) Adhesions
Q169 Gastrointestinal
An 11-year-old boy from Asia develops severe abdominal
pain and signs of SBO. He has no previous surgeries.
Eosinophils = 15%. What is the diagnosis?

A) Ascariasis - endemic to Asia, Africa, SA; often


asymptomatic but may cause pulmonary or intestinal
symptoms; may cause SBO; tx: albendazole
B) Adhesions
Q170 Gastrointestinal
Which vaccine should a patient with a history of
intussusception not receive?

A) Rotavirus
B) Influenza
Q170 Gastrointestinal
Which vaccine should a patient with a history of
intussusception not receive?

A) Rotavirus
B) Influenza
Q170 Gastrointestinal
Which vaccine should a patient with a history of
intussusception not receive?

A) Rotavirus - small risk of intussusception


B) Influenza
Q171 Gastrointestinal
All patients with MALT
lymphomas should be
tested for _.
Q171 Gastrointestinal
All patients with MALT
lymphomas should be
tested for _.

H pylori
Q171 Gastrointestinal
All patients with MALT
lymphomas should be
tested for _.
H pylori plays a major role in the
pathogenesis of extranodal marginal zone
B-cell lymphomas; patients who are
positive and an early stage MALT
lymphoma should undergo H pylori
eradication therapy
Q172 Gastrointestinal
A 76-year-old male is being evaluated for iron
deficiency anemia. A single FOBT is negative. He also
has a systolic murmur at the right intercostal space
with a normal S2. What is the appropriate next step?

A) Echocardiogram
B) Endoscopy and colonoscopy
Q172 Gastrointestinal
A 76-year-old male is being evaluated for iron
deficiency anemia. A single FOBT is negative. He also
has a systolic murmur at the right intercostal space
with a normal S2. What is the appropriate next step?

A) Echocardiogram
B) Endoscopy and colonoscopy
Q172 Gastrointestinal
A 76-year-old male is being evaluated for iron
deficiency anemia. A single FOBT is negative. He also
has a systolic murmur at the right intercostal space
with a normal S2. What is the appropriate next step?

A) Echocardiogram - not severe AS b/c normal S2


B) Endoscopy and colonoscopy - to assess GI bleed
Q173 Gastrointestinal
A 52-year-old woman undergoes colonoscopy - two
small hyperplastic polyps are found and removed. The
patient's mother developed colon cancer at age 81.
What is the best next step?

A) Colonoscopy in 1 year
B) Colonoscopy in 10 years
Q173 Gastrointestinal
A 52-year-old woman undergoes colonoscopy - two
small hyperplastic polyps are found and removed. The
patient's mother developed colon cancer at age 81.
What is the best next step?

A) Colonoscopy in 1 year
B) Colonoscopy in 10 years
Q173 Gastrointestinal
A 52-year-old woman undergoes colonoscopy - two
small hyperplastic polyps are found and removed. The
patient's mother developed colon cancer at age 81.
What is the best next step?

A) Colonoscopy in 1 year
B) Colonoscopy in 10 years
small hyperplastic polyps don't ↑risk for cancer; only first degree relative with
colorectal cancer at age <60 matters; then get screening every 5 years
Q174 Gastrointestinal
A 26-year-old woman comes to the ED with worsening
abdominal pain and intermittent vomiting for the last
day; she has not had a bowel movement in a few
days. Temp = 98.6 F; BP = 90/62. Her abdomen is
distended with hyperactive bowel sounds. What is
most likely present in this patient's history?

A) Appendectomy
B) Weight loss
Q174 Gastrointestinal
A 26-year-old woman comes to the ED with worsening
abdominal pain and intermittent vomiting for the last
day; she has not had a bowel movement in a few
days. Temp = 98.6 F; BP = 90/62. Her abdomen is
distended with hyperactive bowel sounds. What is
most likely present in this patient's history?

A) Appendectomy
B) Weight loss
Q174 Gastrointestinal
A 26-year-old woman comes to the ED with worsening
abdominal pain and intermittent vomiting for the last
day; she has not had a bowel movement in a few
days. Temp = 98.6 F; BP = 90/62. Her abdomen is
distended with hyperactive bowel sounds. What is
most likely present in this patient's history?

A) Appendectomy - adhesions most common SBO


etiology!
Q175 Gastrointestinal
A 35-year-old man presents with abdominal pain,
distention, and the following x-ray. Temp = 98.6 F.
What is the diagnosis?

A) Toxic megacolon
B) Gastric outlet obstruction
C) Cecal volvulus
Q175 Gastrointestinal
A 35-year-old man presents with abdominal pain,
distention, and the following x-ray. Temp = 98.6 F.
What is the diagnosis?

A) Toxic megacolon
B) Gastric outlet obstruction
C) Cecal volvulus
Q175 Gastrointestinal
A 35-year-old man presents with abdominal pain,
distention, and the following x-ray. Temp = 98.6 F.
What is the diagnosis?

A) Toxic megacolon- fever, bloody diarrhea


B) Gastric outlet obstruction -x-ray looks different
C) Cecal volvulus
when cecum and ascending colon twist on their mesentery (can occur in younger
patients often due to a congenital mobile cecum)
Q175 Gastrointestinal
Cecal volvulus management:
-emergency surgery - resection of the volvulized
colonic segment

Sigmoid volvulus management:


-endoscopic detorsion
Q175 Gastrointestinal
Q176 Gastrointestinal
Management of C diff which has progressed to
peritonitis is:

A) Laparotomy
B) Fecal microbiota transplantation
Q176 Gastrointestinal
Management of C diff which has progressed to
peritonitis is:

A) Laparotomy
B) Fecal microbiota transplantation
Q176 Gastrointestinal
Management of C diff which has progressed to
peritonitis is:
A) Laparotomy - indications for surgery are megacolon,
↑serum lactate, unresponsive progression of infection;
surgery to evaluate necrosis/perforation and resect
diseased colon
B) Fecal microbiota transplantation - helps restore
healthy guy microbiome for recurrent CDI
Q176 Gastrointestinal
C diff risk factors
-antibiotics use
-age > 65
-gastric acid suppression (e.g., PPI's)
-less common: IBD, cancer chemotherapy
Q177 Gastrointestinal
Subacute RUQ pain, weight loss, signs of biliary
obstruction (e.g., jaundice), elevated CA-19, and
dilation of the intrahepatic or common bile duct
are seen in:

A) Cholangiocarcinoma
B) HCC
Q177 Gastrointestinal
Subacute RUQ pain, weight loss, signs of biliary
obstruction (e.g., jaundice), elevated CA-19, and
dilation of the intrahepatic or common bile duct
are seen in:

A) Cholangiocarcinoma
B) HCC
Q177 Gastrointestinal
Subacute RUQ pain, weight loss, signs of biliary
obstruction (e.g., jaundice), elevated CA-19, and
dilation of the intrahepatic or common bile duct
are seen in:

A) Cholangiocarcinoma - highly lethal


B) HCC - CA-19 rarely elevated
Q177 Gastrointestinal
Subacute RUQ pain, weight loss, signs of biliary
obstruction (e.g., jaundice), elevated CA-19, and
dilation of the intrahepatic or common bile duct
are seen in:
A) Cholangiocarcinoma - highly lethal
B) HCC - CA-19 rarely elevated - note: AFP
elevated only 50% of time in HCC
Q178 Gastrointestinal
A 77-year-old woman has progressive abdominal pain, nausea/vomiting,
abdominal distention and high-pitched bowel sounds. X-rays show
distended loops of bowel with air fluid levels. She has never had
abdominal surgery before. There is fullness and tenderness within the
right groin. What is the diagnosis?

A) Intussusception
B) SMA occlusion
C) Adhesions
D) Hernia
Q178 Gastrointestinal
A 77-year-old woman has progressive abdominal pain, nausea/vomiting,
abdominal distention and high-pitched bowel sounds. X-rays show
distended loops of bowel with air fluid levels. She has never had
abdominal surgery before. There is fullness and tenderness within the
right groin. What is the diagnosis?

A) Intussusception
B) SMA occlusion
C) Adhesions
D) Hernia
Q178 Gastrointestinal
A 77-year-old woman has progressive abdominal pain, nausea/vomiting,
abdominal distention and high-pitched bowel sounds. X-rays show
distended loops of bowel with air fluid levels. She has never had
abdominal surgery before. There is fullness and tenderness within the
right groin. What is the diagnosis?

A) Intussusception - very rare in adults; groin findings unexpected


B) SMA occlusion - acute onset severe periumbilical pain
C) Adhesions - no history of surgery (or IBD); no groin findings
D) Hernia - femoral hernias can cause SBO
Q179 Gastrointestinal
A 56-year-old woman with a history of
chronic constipation has "horrible"
rectal pain that began yesterday. Rectal
exam shows the following. What is the
next step?

A) Hemorrhoidectomy
B) Stool softeners, sitz baths, &
nitroglycerin cream
Q179 Gastrointestinal
A 56-year-old woman with a history of
chronic constipation has "horrible"
rectal pain that began yesterday. Rectal
exam shows the following. What is the
next step?

A) Hemorrhoidectomy
B) Stool softeners, sitz baths, &
nitroglycerin cream
Q179 Gastrointestinal
A 56-year-old woman with a history of
chronic constipation has "horrible"
rectal pain that began yesterday. Rectal
exam shows the following. What is the
next step?

A) Hemorrhoidectomy - <72 hours


B) Stool softeners, sitz baths, &
nitroglycerin cream - too severe
Q180 Gastrointestinal
An elderly man with a history of metastatic cancer and
chronic constipation now has fecal incontinence (small
volume soft stools). The abdomen is slightly distended. Vital
signs are normal. What is the next step?

A) Check for fecal impaction


B) CT scan
Q180 Gastrointestinal
An elderly man with a history of metastatic cancer and
chronic constipation now has fecal incontinence (small
volume soft stools). The abdomen is slightly distended. Vital
signs are normal. What is the next step?

A) Check for fecal impaction


B) CT scan
Q180 Gastrointestinal
An elderly man with a history of metastatic cancer and
chronic constipation now has fecal incontinence (small
volume soft stools). The abdomen is slightly distended. Vital
signs are normal. What is the next step?

A) Check for fecal impaction - common in older patients with


chronic constipation; backup of stool can lead to incontinence
as liquid stools sneaks around the backup
B) CT scan
Q181 Gastrointestinal
"The Squirt Sign" is seen in:
Q181 Gastrointestinal
"The Squirt Sign" is seen in:

Hirschsprung disease
Q181 Gastrointestinal
"The Squirt Sign" is seen in:
Hirschsprung disease
Classic rectal exam in patients with Hirschsprung
disease reveals an expulsion of stool or gas ("squirt
sign") due to temporary relief of the distal
obstruction.
Q182 Gastrointestinal
A 2-day old girl has abdominal distention and has not yet
had a bowel movement. After imaging is taken, urgent
laparotomy is performed in which sticky stool is evacuated
from the illeum. What is this condition associated with?

A) Nasal polyps
B) Duodenal atresia
Q182 Gastrointestinal
A 2-day old girl has abdominal distention and has not yet
had a bowel movement. After imaging is taken, urgent
laparotomy is performed in which sticky stool is evacuated
from the illeum. What is this condition associated with?

A) Nasal polyps
B) Duodenal atresia
Q182 Gastrointestinal
A 2-day old girl has abdominal distention and has not yet
had a bowel movement. After imaging is taken, urgent
laparotomy is performed in which sticky stool is evacuated
from the illeum. What is this condition associated with?

A) Nasal polyps - meconium illeus is virtually diagnostic of


cystic fibrosis
B) Duodenal atresia - meconium is not sticky in DS
Q183 Gastrointestinal
Which marker in acute hepatitis correlates with an
increased likelihood of mortality

A) bilirubin
B) PT
C) ALT
D) ALP
Q18 Gastrointestinal
3
Which marker in acute hepatitis correlates with an
increased likelihood of mortality

A) bilirubin
B) PT
C) ALT
D) ALP
Q18 Gastrointestinal
3
Which marker in acute hepatitis correlates with an
increased likelihood of mortality

A) bilirubin
B) PT - liver not working! not making clotting factors!
C) ALT
D) ALP
Q184 Gastrointestinal
Which will be abnormal first after aquiring HBV infection?

A) bilirubin
B) e-antigen
C) surface antigen
D) core IgM antibody
Q184 Gastrointestinal
Which will be abnormal first after acquiring HBV
infection?

A) bilirubin
B) e-antigen
C) surface antigen
D) core IgM antibody
Q184 Gastrointestinal
Which will be abnormal first after acquiring HBV
infection?

A) bilirubin
B) e-antigen
C) surface antigen measures actual viral particles; the
other choices measure the body's response
D) core IgM antibody
Q18 Gastrointestinal
5
Which indicates that a pregnant woman will transmit
infection to her child?

A) bilirubin
B) e-antigen
C) surface antigen
D) core IgM antibody
Q18 Gastrointestinal
5
Which indicates that a pregnant woman will transmit
infection to her child?

A) bilirubin
B) e-antigen
C) surface antigen
D) core IgM antibody
Q18 Gastrointestinal
5
Which indicates that a pregnant woman will transmit
infection to her child?

A) bilirubin
B) e-antigen - indicates transmissibility
C) surface antigen
D) core IgM antibody
Q18 Gastrointestinal
6
A 27-year-old woman falls during gymnastics
and now has severe abdominal pain. On day 3,
she has diffuse abdominal pain and nausea.
The abdomen is distended and tender. Xray is
shown. What is the diagnosis?

A) Cecal volvulus
B) Paralytic ileus
C) Sigmoid volvulus
D) Perforated viscus
Q18 Gastrointestinal
6
A 27-year-old woman falls during gymnastics
and now has severe abdominal pain. On day 3,
she has diffuse abdominal pain and nausea.
The abdomen is distended and tender. Xray is
shown. What is the diagnosis?

A) Cecal volvulus
B) Paralytic ileus
C) Sigmoid volvulus
D) Perforated viscus
Q18 Gastrointestinal
6 volvulus
A) Cecal - uncommon in trauma;
transition point at the sigmoid colon
B) Paralytic ileus - most common after
abdominal surgery, but can also be seen in
trauma; xray shows uniform dilation, gas filled
loops of bowel; tx: bowel rest, supportive care
C) Sigmoid volvulus - uncommon in trauma;
transition point at the sigmoid colon
D) Perforated viscus - would show air under the
diaphragm
Q18 Gastrointestinal
7
Which of the following benign liver lesions grows with
estrogen?

A) Focal nodular hyperplasia


B) Hemangioma
C) Hepatic adenoma
Q18 Gastrointestinal
7
Which of the following benign liver lesions grows with
estrogen?

A) Focal nodular hyperplasia


B) Hemangioma
C) Hepatic adenoma
Q18 Gastrointestinal
7
Which of the following benign liver lesions grows with
estrogen?
A) Focal nodular hyperplasia
"central stellate scarring", no malignant potential
B) Hemangioma
"tuft of abnormal vessels"; no malignant potential
C) Hepatic adenoma
grows with estrogen; may cause pain; tiny risk of malignancy
Q18 Gastrointestinal
8
Post-gastrectomy, a man develops nausea, diarrhea,
cramps, and diaphoresis. What is the pathogenesis?

A) Pyloric sphincter dysfunction


B) Gram-negative infection
Q18 Gastrointestinal
8
Post-gastrectomy, a man develops nausea, diarrhea,
cramps, and diaphoresis. What is the pathogenesis?

A) Pyloric sphincter dysfunction


B) Gram-negative infection
Q18 Gastrointestinal
8
Post-gastrectomy, a man develops nausea, diarrhea,
cramps, and diaphoresis. What is the pathogenesis?

A) Pyloric sphincter dysfunction -


Dumping syndrome is a common postgastrectomy complication (occurring in up to 50%
of patients!); PS dysfunction leads to rapid emptying of hypertonic gastric contents into
the duodenum and SI, leading to fluid shift into the SI, leading to hypotension and
stimulation of autonomic reflexes; dietary recommendations: small frequent meals, eat
slowly, avoid simple sugars, increase fiber, fluids between meals rather than during
Q18 Gastrointestinal
9
How is colon adenocarcinoma that has metastasized to
the liver treated?

A) Surgical resection
B) Chemotherapy
C) Palliative care
Q18 Gastrointestinal
9
How is colon adenocarcinoma that has metastasized to
the liver treated?

A) Surgical resection
B) Chemotherapy
C) Palliative care
Q18 Gastrointestinal
9
How is colon adenocarcinoma that has metastasized to
the liver treated?

A) Surgical resection - liver is the most common location


of colon cancer metastasis (b/c the colon venous system
drains into the portal circulation)
B) Chemotherapy - only if unresectable or severe metastasis
C) Palliative care - only if surgery isn't an option due to poor health
state
Q19 Gastrointestinal
A 0
newborn has difficulty feeding, choking, as well
copious oral secretions. What is the next step?

A) Echocardiogram
B) Nasogastric tube
Q19 Gastrointestinal
A 0
newborn has difficulty feeding, choking, as well
copious oral secretions. What is the next step?

A) Echocardiogram
B) Nasogastric tube
Q19 Gastrointestinal
A 0
newborn has difficulty feeding, choking, as well
copious oral secretions. What is the next step?

A) Echocardiogram
B) Nasogastric tube - diagnosis for tracheo-esophageal
fistula with esophageal atresia (congenital but often not
detected until shortly after birth when baby develops
copious saliva & choking, coughing, and/or vomiting after
feeding)
Q19 Gastrointestinal
0
Q19 Gastrointestinal
A 1
35-year-old Down Syndrome patient who suddenly
develops constipation should be evaluated for:

A) Aganglionic megacolon
B) Thyroid function studies
Q19 Gastrointestinal
A 1
35-year-old Down Syndrome patient who suddenly
develops constipation should be evaluated for:

A) Aganglionic megacolon
B) Thyroid function studies
Q19 Gastrointestinal
A 1
35-year-old Down Syndrome patient who suddenly
develops constipation should be evaluated for:

A) Aganglionic megacolon
B) Thyroid function studies - in general, new onset
constipation may indicate an underlying medical
condition; this is esp. true in DS patients who are at risk
for endocrine abnormalities
Q19 Gastrointestinal
2
What is the treatment for asymptomatic sliding hiatal
hernia (e.g., found incidentally)?

A) H2 blockers
B) Steroids
C) No treatment
Q19 Gastrointestinal
2
What is the treatment for asymptomatic sliding hiatal
hernia (e.g., found incidentally)?

A) H2 blockers
B) Steroids
C) No treatment
Q19 Gastrointestinal
2
What is the treatment for asymptomatic sliding hiatal
hernia (e.g., found incidentally)?

A) H2 blockers
B) Steroids
C) No treatment - only treat if patient has reflux
symptoms (e.g., PPI's)
Q19 Gastrointestinal
3
Bloating, flatulence, abdominal discomfort, and watery
diarrhea in a patient with ileocecal resection is consistent
with _.

A) Lactose intolerance
B) Small intestinal bacterial overgrowth
Q19 Gastrointestinal
3
Bloating, flatulence, abdominal discomfort, and watery
diarrhea in a patient with ileocecal resection is consistent
with _.

A) Lactose intolerance
B) Small intestinal bacterial overgrowth
Q19 Gastrointestinal
3
Bloating, flatulence, abdominal discomfort, and watery
diarrhea in a patient with ileocecal resection is consistent
with _.
B) Small intestinal bacterial overgrowth -
normally SI bacterial growth is inhibited by gastric acid and PB juices as well
as peristalsis; in addition, ileocecal valve serves as a physical barrier to
prevent colonic bacteria from entering the SI; therefore, SIBO can develop in
motility disorders or after ileocecal resection; dx: jejunal aspirate (gold
standard but invasive) or carbohydrate breath test (to detect fermentation in
the small bowel, indicating SIBO)
Q19 Gastrointestinal
4
How is severe dehydration treated?

A) Oral rehydration
B) Dextrose
C) 1/2 normal saline IV
D) Normal saline IV
Q19 Gastrointestinal
4
How is severe dehydration treated?

A) Oral rehydration
B) Dextrose
C) 1/2 normal saline IV
D) Normal saline IV
Q19 Gastrointestinal
4
How is severe dehydration treated?

A) Oral rehydration - mild dehydration


B) Dextrose - good for low blood sugar
C) 1/2 normal saline IV - hypotonic
D) Normal saline IV
Q19 Gastrointestinal
5
Fever, RUQ pain, nausea/vomiting, air-fluid levels
in gallbladder, gas in gallbladder wall, and normal
aminotransferases =

A) Acute cholangitis
B) Emphysematous cholecystitis
Q19 Gastrointestinal
5
Fever, RUQ pain, nausea/vomiting, air-fluid levels
in gallbladder, gas in gallbladder wall, and normal
aminotransferases =

A) Acute cholangitis
B) Emphysematous cholecystitis
Q19 Gastrointestinal
5
Fever, RUQ pain, nausea/vomiting, air-fluid levels
in gallbladder, gas in gallbladder wall, and normal
aminotransferases =

A) Acute cholangitis - dilated bile ducts instead of gas


B) Emphysematous cholecystitis - a life-threatening form of
acute cholescystitis due to infection with gas-forming
bacteria (Clostridium)
Q19 Gastrointestinal
5
Fever, RUQ pain, nausea/vomiting, air-fluid levels
in gallbladder, gas in gallbladder wall, and normal
aminotransferases =

A) Acute cholangitis - dilated bile ducts instead of gas


B) Emphysematous cholecystitis -
The treatment of choice is emergency cholecystectomy, along with broad-
spectrum antibiotics (eg, piperacillin-tazobactam) and bile culture.
Q19 Gastrointestinal
6
What is seen in this CT scan?
Q19 Gastrointestinal
6
What is seen in this CT scan?

Retroperitoneal
Hematoma!
Q19 Gastrointestinal
7
A patient has signs of SBO and xray is shown.
What is the next step?

A) Emergency surgery
B) Barium enema
C) Nasogastric tube
Q19 Gastrointestinal
7
A patient has signs of SBO and xray is shown.
What is the next step?

A) Emergency surgery
B) Barium enema
C) Nasogastric tube
Q19 Gastrointestinal
7
A) Emergency surgery - subdiaphgram
free air indicates perforated viscus
B) Barium enema - contraindicated if
perforation is suspected; generally used
for diagnosis of functional or structural
abnormalities
C) Nasogastric tube - for decompressing
SBO, but here we need to intervene with
surgery!
Q19 Gastrointestinal
8
What should be done in a patient with epigastric
abdominal pain, nausea, and elevated lipase, but
no gallstones?

A) IV fluids and supportive care


B) ERCP
Q19 Gastrointestinal
8
What should be done in a patient with epigastric
abdominal pain, nausea, and elevated lipase, but
no gallstones?

A) IV fluids and supportive care


B) ERCP
Q19 Gastrointestinal
8
What should be done in a patient with epigastric
abdominal pain, nausea, and elevated lipase, but
no gallstones?
A) IV fluids and supportive care - tx: for acute
pancreatitis
B) ERCP - for sphincterotomy and stone removal in
patients with gallstone pancreatitis
Q19 Gastrointestinal
9
A patient recovers from a cardiac arrest and is now
asymptomatic. His aminotransferase levels are
severely elevated (>1200). What is the cause?

A) Acalculous cholecystitis
B) Heptacyte ischemia
Q19 Gastrointestinal
9
A patient recovers from a cardiac arrest and is now
asymptomatic. His aminotransferase levels are
severely elevated (>1200). What is the cause?

A) Acalculous cholecystitis
B) Heptacyte ischemia
Q19 Gastrointestinal
9
A patient recovers from a cardiac arrest and is now asymptomatic.
His aminotransferase levels are severely elevated (>1200). What
is the cause?

A) Acalculous cholecystitis - a complication of shock or


critical illness; extreme elevation of aminotransferases not
expected
B) Heptacyte ischemia - necrosis in zone 3; poor
prognosis; in patients survive, liver recovers
Q200 Gastrointestinal
How is pyloric stenosis diagnosed?

A) Abdominal x-ray
B) Abdominal ultrasound
Q200 Gastrointestinal
How is pyloric stenosis diagnosed?

A) Abdominal x-ray
B) Abdominal ultrasound
Q200 Gastrointestinal
How is pyloric stenosis diagnosed?

A) Abdominal x-ray - for SBO


B) Abdominal ultrasound - (a classic olive-shaped
abdominal mass may be palpable); ultrasound
shows a thick and elongated pyloris; tx:
pyloromyotomy

You might also like